SlideShare a Scribd company logo
1 of 178
ACL Tear & Associations
īŽ 38 yo M, hurt skiing
Initial findings? Justification for further
imaging?
Additional Imaging
Findings on T1 image?
Findings? What additional abnormalities are suspected, if any?
īŽ T1W1 FSE Fat sat (farther
lateral)
Findings so far? Any additional concern (s)?
Representative Images in the Notch
Any other injury you should specifically look for?
Additional Findings (separate patient)
Summery of all findings & Dx for this case?
ACL Tear & Associations
īŽ Initial (film) images most importantly showed
an avulsion fracture from the lateral portion of
the proximal tibia. Marked bone edema at this
site was confirmed on the MR images.
īŽ This is Segond Fracture, and represents an
avulsion of the lateral capsular ligament. It
occurs posterior to Gerdy’s tubercle.
īŽ It is important to diagnose this fracture
because it is associated with an ACL tear in
75-100%, and a meniscal tear in 67%!
īŽ Additional MR images demonstrate “Kissing
Contusions” in the lateral portions of the distal femur
and proximal tibia. Seen in > 50% of ACL tears
(specific but not highly sensitive).
īŽ Images through intercondylar notch show a large
amount of fluid/edema with non-visualization of the
ACL: c/w tear.
īŽ Other indirect signs (not shown) include anterior
displacement of the tibia (anterior drawer sign) and
PCL bowing.
īŽ The ACL is the most commonly injured of the major
ligaments in the knee.
Related Findings
īŽ Final images for this case (from a different
patient) showed a tear of the medial collateral
ligament & the medial meniscus.
īŽ In conjunction with an ACL tear, this
represents the “Terrible Triad of O’donoghue”
īŽ Classically seen as “clipping injury” from
football (valgus/abduction type injury)
Maisonneuve Fracture
īŽ 35 yo Female, s/p fall; ankle pain
Findings, Dx? Any Further Imaging
Needed/ Recommended?
Additional images
Final Dx? Mechanism of Injury?
īŽ Maisonneuve Fracture
īŽ 1st set of images showed ankle fractures involving the medial
malleolus and posterior tibia. However, the widening between
the distal tibia and fibula should also be noted (suggests tear of
the interosseus membrane).
īŽ Images of the proximal tibia confirm a spiral fracture of the
proximal 3rd of the fibula (interosseus membrane is always torn
to this point)
īŽ Mechanism: Abduction and External rotation (forces talus
laterally against the fibula); distal tibiofibular syndesmosis
always ruptured
īŽ Associated with: rupture of the deltoid ligament, one candidate
initially shown only widening of ankle joint space.
Sarcoid of Bone
īŽ 63 year old Female, Hand pain
Findings?
Same patient - Foot
Findings, DDx?
Same patient after 2 yrs. Rx
Try to give Dx/ DDx before Continuingâ€Ļ
Additional Images Found In Patient’s Javcket- “Classic”
Summary of Findings and Final Dx?
īŽ Sarcoid of Bone
īŽ Initial images showed a reticulated, “lacelike” pattern involving
the 2nd and 5th digits of the left hand, and the 3rd and 5th digits
of the right hand. Foot image showed well-defined cystic/
lacelike lesions in the distal phalanx of the 1st digit.
īŽ Mineralization and joint spaces essentially normal; no erosions
īŽ Lesions improved after systemic treatment (MTX & Steroids)
īŽ CXR & MRI confirmed sarcoid findings in the chest & brain
īŽ Can also have neuropathy-like destruction of distal phalanges
(not present in this case)
īŽ Osseus involvement seen in 6-20% of sarcoid cases.
Case 1
īŽ History
īŽ The patient fell on an outstretched hand and
now complains of pain at the thumb.
īŽ Diagnosis
īŽ Gamekeeper’s Thumb or Skier’s Thumb.
īŽ Findings
īŽ Soft tissue swelling is noted about the base of
the thumb. There is a small, mildly displaced
fracture fragment from
īŽ the ulnar aspect of the base of the proximal
phalanx of the first digit.
īŽ Discussion
īŽ Gamekeeper’s or Skier’s Thumb results from disruption of the
ulnar collateral ligament (UCL) secondary to a valgus
īŽ stress applied at the first metacarpophalangeal joint. Abduction-
stress films of the thumb with UCL disruption show a
īŽ first metacarpal-phalangeal angle of greater than 30 degrees.
Often there is an associated avulsed bone fragment off
īŽ the base of the proximal phalanx as in the current case.
īŽ A “Stener Lesion” occurs when the torn end of the UCL
becomes displaced superficial to the adductor pollicus
īŽ aponeurosis. MRI and ultrasound best evaluate UCL
entrapment.
Case 2
īŽ History
īŽ Pain. Evaluate for possible fracture.
īŽ Diagnosis
īŽ Polyostotic Fibrous Dysplasia.
īŽ Findings
īŽ Plain film of the pelvis shows a “ Shepherd’s Crook”
deformity of the femoral neck. There is a long
segment expansile lesion of the proximal femur with
endosteal scalloping and
īŽ a ground glass matrix. A second geographic lesion
with a thin sclerotic margin, a short transition zone,
and a ground glass matrix is noted in the left pelvis.
Discussion
īŽ Fibrous dysplasia is a developmental dysplasia where bone is displaced by
fibrous tissue. Irregularly arrayed trabeculae of immature bone formed by a
metaplasia of fibrous
īŽ tissue may be present. Fibrous dysplasia may involve one bone (monostotic, 80
percent) or multiple bones (polyostotic, 20 percent). The cause of fibrous
dysplasia is unknown
īŽ and is not hereditary.
īŽ Polyostotic fibrous dysplasia has a predilection for involvement of one side of
the body seen in 90 percent of cases. Polyostotic fibrous dysplasia may be
associated with
īŽ McCune – Albright Syndrome and other endocrine anomalies. Sarcomatous
transformation is rare.
īŽ Radiographic findings include expansile lesions that may exhibit endosteal
scalloping. The replacement of bone by fibrous tissue leads to a “Smokey” or
“ground glass”
īŽ appearance to the lesions. Some lesions may appear “cystic” with a thin
sclerotic rim as in the current case. Bones are prone to fracture. Fractures of the
femoral neck may
īŽ contribute to the classic “ Shepherd’s Crook” deformity.
Case 3
īŽ History
īŽ 24 year old male athlete with anterior knee
pain.
Diagnosis
īŽ Jumper’s Knee (Infrapatellar Insertional
Tendinosis).
Findings
īŽ Plain Images show thickening and abnormal
high T2 signal in the proximal patellar tendon
at its origin on the inferior patella
īŽ Discussion
īŽ Patellar tendinosis (Jumper’s Knee) is most
commonly found in athletes primarily engaged in
running and jumping activities. Jumper’s Knee is a
chronic functional overload
īŽ injury with both micro- and partial macro-tearing of
the tendon. MRI demonstrates thickening and high
T2 signal with the proximal patellar tendon. Surgery
is generally reserved
īŽ for those patients with inadequate symptom relief
from conservative therapy
Case 4
īŽ History
īŽ Pain at forefoot
īŽ Diagnosis
īŽ Freiberg’s Infraction.
īŽ Findings
īŽ There is flattening, fragmentation and sclerosis
involving the third metatarsal head. There is cortical
thickening of the adjacent meta-diaphyseal region of
the metatarsal. The
īŽ adjacent proximal phalangeal exhibits irregularity,
broadening and sclerosis at its base.
īŽ Discussion
īŽ Freiberg’s infraction affects women three to four times more
commonly than men and is most commonly seen in adolescent
girls ages 13 to 18 years. The second metatarsal
īŽ head is most commonly involved and experimental studies have
indicated that this digit is most vulnerable secondary to its
relative length. The third and fourth metatarsals may
īŽ be involved, but the first digit is rarely affected.
īŽ Pathogenesis remains speculative but likely involves repetitive
trauma as the inciting event. Pathologically, osteonecrosis is
present. The radiographic findings are virtually
īŽ pathognomonic and consistent with osteonecrosis at a
characteristic site.
A 52- years female presented with
backache
īŽ The lesion is largely, but not solely,
geographic in that it appears more
permeative in the more anterior portion of the
vertebral body on the lateral film. It does not
have a sclerotic margin and there is not
matrix calcification. there has been slightly
compression of the vertebral body in both the
AP and medial lateral dimension. While this
widening could be accounted for the slight
collapse of the body, another explanation may
be reasonable as well.
īŽ Film. 3
īŽ They shows abnormal osseous uptake at several
sites, including T11, L2, L5, the right acetabular
region, right hip and femoral neck, right radius, and
patchy abnormal uptake in the skull. This information
is immediately useful in eliminating several entities in
our differential diagnosis: chordoma, giant-cell tumor,
osteoblastoma, and lymphoma. Of the remaining
three, the pattern of bones that is involved is most
characteristic of Paget’s disease.
īŽ Film4.
īŽ This film demostrates that the radius is
slightly overgrown and bowed. In addition,
there is slight cortical thickening. These are
subtle findings, but are so characteristic of
Paget’s disease.
īŽ We see lytic highly destructive lesions involving the
right femoral metadiaphyseal region, both iliac wings,
and the left humeral epiphysis. All of the lesions are
aggressive, showing a permeative pattern and wide
zone of transition without margination. Although of
plains films alone in this case are not sufficiently
specific to yield the histologic diagnosis, you shows
now be able to supply a reasonable differential
diagnosis. Remember also that we like to use all of
the information supplied by the films, as this patient is
female.
īŽ Although this is a subtle example of early Paget's disease, it should be
remembered that in its early phase Paget's disease can appear lytic
and moderately aggressive. This case emphasizes three points. First
of all, it was important to categorize the spine lesion as aggressive or
moderately aggressive rather than malignant, since the latter
categorization would have precluded Paget's disease from the
differential diagnosis. Secondly, following the algorithmic approach to
the workup paid off in this case as it does in most cases. Obtaining the
bone scan and with it the information regarding the polyostotic nature of
the lesion limited the differential diagnosis. Finally, the bone scan
directs your attention to other areas to be imaged with plain film. This
will very often lead to confirmation of a diagnosis. In this case, CT or
MR of the original lesion would not have added significantly to our
diagnostic capabilities, since they would have confirmed the presence
of a moderately aggressive lesion. Following the algorithmic approach
saved the patient the time, expense, and discomfort of undergoing
these other testing procedures, as well as the discomfort and anxiety of
a biopsy.
Diagnosis
īŽ Introduction of Musculoskeletal Tumors.
īŽ The radiologic workup of musculoskeletal tumors can be both cost-
efficient and extremely helpful to the referring clinician if one proceeds
in a thoughtful and logical manner. The algorithm shown on Diagram 1
is one that we have found useful for aggressive osseous lesions.
īŽ Initially, a musculoskeletal tumor should be simply imaged with a plain
film. It should be remembered that plain films remain the most reliable
imaging method for assessment of both biologic activity and probable
histologic diagnosis of an osseous lesion. Although soft tissue
involvement by an osseous lesion may be incompletely assessed by
plain film, the osseous findings are seen with much better resolution on
plain radiographs than with either CT or MR. Plain film therefore is
used to arrive at a reasonable differential diagnosis or at least to
categorize the lesion as to degree of aggressiveness. In order to do
this, we have found it useful to include an assessment of 10
determinants in the description of a tumor. If these determinants are
accurately described, the correct diagnosis or at least a limited
differential diagnosis usually becomes obvious. These determinants
are as follows:
īŽ 1. Age of the patient. This can be an extremely important
determinant in some lesions in which the age range of
occurrence may be quite narrow. For example, malignant
osseous lesions in patients under one year of age are usually
metastatic neuroblastoma. Malignant osseous lesions in the
age range of 1 to 30 are usually osteosarcoma or Ewing's
sarcoma. Malignant osseous lesions in the 30- to 60-year range
most commonly will be either chondrosarcoma, primary
lymphoma, or malignant fibrous histiocytoma, while malignant
lesions in the age range over 50 most commonly will be due to
metastatic disease or multiple myeloma. Several other osseous
lesions have fairly limited age ranges as well. These will be
discussed with the individual cases later on in the section.
īŽ 2. Soft tissue involvement. Cortical breakthrough of a bone lesion to create a
soft tissue mass generally suggests an aggressive lesion. Such soft tissue
masses will often distort but not obliterate nearby muscle planes.
īŽ 3. Pattern of bone destruction. Common terminology includes the terms
"geographic" (well-defined or map-like lesion, the least aggressive pattern),
"moth-eaten" (holes, with less well-defined margins, appearing more
aggressive), and "permeative" (a poorly demarcated pattern which is often very
difficult to visualize and represents a highly aggressive lesion). It is not always
easy to differentiate between the moth-eaten and permeative patterns.
Furthermore, since both represent an aggressive pattern, it is not necessary to
differentiate between the two, and the term permeative should serve well for
both.
īŽ 4. Size of lesion. Generally, a larger lesion (greater than 5 cm) is more likely to
be malignant or aggressive, but there are many exceptions to this statement,
and other determinants are generally more important than this one.
īŽ 5. Location of the lesion. Three different types of locations should be
noted: the particular bone that is involved, the location in a transverse
axis, and the location in a longitudinal axis of a long bone.
Occasionally the particular bone involved may be important to the
diagnosis. One such example is the tibia, which in addition to hosting
most tumors that one can think of, also is the most common location for
three uncommon tumors, adamantinoma, ossifying fibroma, and
chondromyxoid fibroma. Other categorizations of particular bone
involvement might be useful such as axial versus appendicular or flat
versus tubular bones, with many lesions clearly favoring one over the
other. It is worthwhile to categorize a lesion's location in the transverse
axis of a tubular bone (central, eccentric, or a cortically-based
epicenter). As will be noted in discussions of individual tumors later on,
many tumors have very characteristic locations in the transverse axis.
Similarly, many tumors have characteristic locations along the long axis
of a tubular bone (epiphysis, metaphysis or diaphysis). Therefore, this
location should be identified in the description of the osseous lesion as
well.
īŽ 6. Zone of transition of the lesion from abnormal to normal bone. A
wide zone of transition denotes an aggressive lesion, while a narrow
zone is a much less aggressive lesion.
īŽ 7. Margination of the lesion. A sclerotic margin generally represents a
nonaggressive lesion whereas a nonsclerotic margin often represents
an aggressive lesion. There are, however, important exceptions to this,
including giant-cell tumor and enchondroma. It is generally true that
the determinants narrow zone of transition and sclerotic margin occur
together in a lesion and suggest that it is nonaggressive. However,
these terms are not synonymous. Similarly, the determinants wide
zone of transition and nonsclerotic margin usually occur together in a
lesion and suggest that it is aggressive. However, occasionally one
may see a lesion with a narrow zone of transition but no sclerotic
margin. This very unusual combination of determinants is found most
commonly in giant-cell tumor and less commonly in plasmacytoma. It
is therefore useful to describe these determinants separately.
īŽ 8. Presence of visible tumor matrix. The character of
any tumor matrix should be described, since it may
be tumor specific or may at least allow categorization
of a lesion as bone producing versus cartilage
producing. In general, aggressive bone-forming
tumors produce amorphous osteoid which is often
less dense than normal bone. Less aggressive bone-
forming tumors produce better organized, denser
bone. The matrix of cartilage- producing tumors is
usually quite distinctive, appearing stippled and more
dense than normal bone.
īŽ 9. Host response. An aggressive lesion may not
allow a host response, demonstrating cortical
destruction and penetration, with or without periosteal
reaction. A less aggressive lesion may result in
cortical thickening or sclerosis, cortical thinning
without reactive bone formation, or cortical
expansion. It might be noted that the character of
periosteal reaction is not always a reliable sign in
determining the aggressiveness of the lesion.
However, generally thin linear periosteal reaction is
seen in less aggressive lesions while sunburst
periosteal reaction is seen in the more aggressive
categories.
īŽ 10. Polyostotic versus Monostotic. This is the last
determinant and might be the most important, since
polyostotic lesions automatically restrict the number
of disease processes that might be considered. For
example, nonaggressive polyostotic lesions should
be confined to fibrous dysplasia, Paget's disease,
histiocytosis, multiple exostosis, and multiple
enchondromatosis. Aggressive polyostotic lesions
would be confined to osseous metastases, multiple
myeloma, primary bone tumor with osseous
metastases, an aggressive phase of Paget's disease,
multifocal osteomyelitis, aggressive histiocytosis, and
multifocal vascular bone tumors.
īŽ The description of these 10 determinants should yield the
diagnosis or at least a short differential diagnosis. The
individual musculoskeletal tumors often have very characteristic
features among these determinants. After several introductory
cases, these sets of determinants will be discussed with each
individual entity. If one cannot give a diagnosis, it is important to
conclude with the observation of whether the lesion is
aggressive or nonaggressive rather than malignant or benign.
The reasoning here is that some malignant lesions may appear
nonaggressive and several benign lesions often appear highly
aggressive (especially osteomyelitis and histiocytosis). If one
uses the term benign or malignant in one's description,
consideration of such lesions which may appear aggressive but
act benign often will be precluded.
īŽ If the exact diagnosis is not reached after examination of the plain film,
one might attempt to place the lesion in one of the following five
categories:
īŽ 1. An asymptomatic, benign leave-me-alone lesion, which requires no
further imaging or attention. An example might be a fibrous cortical
defect or classic nonossifying fibroma.
īŽ 2. An asymptomatic, almost certainly benign lesion. Such a lesion
could be safely followed without further workup and an example might
be a large nonossifying fibroma.
īŽ 3. A benign symptomatic lesion with a highly probable diagnosis with
the region of involvement well seen. Examples of this might be a giant-
cell tumor or chondroblastoma. In many cases, one can proceed to
definitive treatment without further imaging of these lesions.
īŽ 4. A lesion of uncertain diagnosis and a mixture of
aggressive and less aggressive features such that
benign or malignant status cannot be confidently
assessed. Although a good radiologist can attempt to
keep the number of lesions assigned to this category
small, some lesions truly belong to this category
(such as a low-grade intermedullary chondrosarcoma
or an aggressive giant-cell tumor) and must be
worked up carefully as if they truly belong to the most
aggressive category.
īŽ 5. An obviously malignant lesion which requires
further workup, perhaps for diagnosis but certainly for
staging.
īŽ The algorithm on Diagram .1 and .1a is for
aggressive osseous lesions (category 4 or 5 above).
Reasoning for the order of examinations is developed
over the next few cases, but note that CT or MR
imaging should generally not be performed until late
in the workup.
īŽ The algorithm for soft tissue musculoskeletal tumors
is different (Diagram .2). Radionuclide scans are not
useful. Furthermore, MR does not reliably
differentiate malignant from benign tumors.
Therefore, biopsy might reasonably be performed
prior to chest CT to determine whether there is even
any need for a metastatic workup.
A year old female who appears cachectic and ill. Look at all the images.
This is a case of polyostotic aggressive lesions in a child. We see lytic
highly destructive lesions involving the right femoral metadiaphyseal
Region, both iliac wings, and the left humeral epiphysis. All the lesions
are aggressive, showing a permeative pattern and wide zone of
transition without margination. Although the plain films alone in this
case are not sufficiently specific to yield the histologic diagnosis, you
should now be able to supply a reasonable differential diagnosis.
remember also that we like to use all of the information supplied by the
films, including that on the Name plate. Although it is not included on
this film, I will tell you that the patient’s first name is Ebony and she is
from New York City. Does this information help you to narrow down the
differential diagnosis?
īŽ Plain films are the only radiographic workup required in this patient prior to
biopsy. CT or MR would not add enough additional information to make their
costs worthwhile. Aggressive polyostotic lesions in a 5-year-old most typically
represent Ewing's sarcoma with osseous metastases or else metastatic
neuroblastoma. However, one must also remember that young patients
occasionally do develop multifocal osteomyelitis which can appear quite
aggressive; the patient's clinical appearance is that of a systemic illness. In
addition, another polyostotic lesion in young patients is eosinophilic granuloma
(EG) or the other diseases in the spectrum of histiocytosis, such as Hand-
Christian-Schuller disease. EG can occasionally appear this aggressive and
definitely deserves to be considered in this case.
īŽ There are two unusual features in this case which make it all the more
interesting. First of all, the lesion in the humerus is centered in the epiphysis.
Epiphyseal lesions are extremely rare and most commonly represent
chondroblastomas or osteomyelitis when seen in the skeletally immature patient.
This obviously is not a chondroblastoma because of its degree of
aggressiveness. One must make the decision to regard the location of this
particular lesion as a red herring and not allow this site of lesion to alter the rest
of the impression of an aggressive polyostotic lesion.
īŽ The second item of interest is the patient's name. The name Ebony gives us the
impression that this patient is black. This information helps one narrow the
diagnosis down. First of all, it strengthens the possible diagnosis of multifocal
osteomyelitis since if this patient has sickle cell anemia, osteomyelitis is a
common complication. However, blood tests showed that the patient is not a
sickler. The fact that the patient is black also serves to virtually eliminate the
possibility that this is a Ewing's sarcoma, since the latter disease is nearly
nonexistent in the black population. Therefore, the most likely diagnosis in this
case is metastatic neuroblastoma. Appropriate testing was performed and the
patient did not have a neuroblastoma. At biopsy, a round-cell lesion was found.
It turned out with further testing to be an unusual lymphoma.
īŽ Narrowing the case down to this particular diagnosis is probably not possible in
this case, but the discussion surrounding it is what makes it interesting.
Furthermore, it serves as a reminder that an aggressive lesion in this age group
should have all of the round-cell lesions included in the differential diagnosis:
metastatic neuroblastoma, Ewing's sarcoma (or other lymphoma), osteomyelitis,
and eosinophilic granuloma. This wide spectrum of diseases including both
benign and malignant lesions is an important differential diagnosis to remember.
īŽ Look first only at film. 1. the radiographic findings are subtle in this case. We see a highly
permeative lesion involving the epiphyseal, metaphyseal and proximal diaphyseal portion of
the left humerus. This lesion has a wide zone of transition and is not marginated. It has
elicited aggressive periosteal reaction both in a linear and spiculated pattern. There is a
large soft tissue mass. The most important feature of this lesion is the presence of osteoid
tumor matrix both within the bone and, in a highly amorphous pattern, within the soft tissue
mass (Film. 1a). What is your diagnosis?
The only possible diagnosis for this extremely aggressive lesion which produces tumor osteoid is
osteosarcoma. How should workup proceed? Surgical staging must be considered next. An
important part of the staging process is determining the presence or absence of metastatic
disease. Most musculoskeletal sarcomas are disseminated by the vascular system, and
therefore metastatic disease is expected to localize in the lungs and, a little less commonly,
in other osseous sites. The algorithm therefore calls for a bone scan to be evaluated next for
the presence of metastatic disease. Look next at film. 2. Give the diagnosis and your
suggestive for the next item in an orderly staging workup.
īŽ Abbreviations used:
īŽ ABC = Aneurysmal bone cyst
īŽ CMF = Chondromyxoid fibroma
īŽ EG = Eosinophilic Granuloma
īŽ GCT = Giant cell tumour
īŽ FD = Fibrous dysplasia
īŽ HPT = Hyperparathyroidism with Brown tumor
īŽ NOF = Non Ossifying Fibroma
īŽ SBC = Simple Bone Cyst
īŽ The most important determinators in the analysis of a potential
bone tumor are:
īŽ The morphology of the bone lesion on a plain radiograph
īŽ Well-defined osteolytic
īŽ ill-defined osteolytic
īŽ Sclerotic
īŽ The age of the patient
īŽ
It is important to realize that the plain radiograph is the most
useful examination for differentiating these lesions.
CT and MRI are only helpful in selected cases.
īŽ In this article there are links to other articles about bone tumors.
īŽ Most bone tumors are osteolytic.
The most reliable indicator in determining whether
these lesions are benign or malignant is the zone of
transition between the lesion and the adjacent normal
bone (1).
Once we have decided whether a bone lesion is
sclerotic or osteolytic and whether it has a well-
defined or ill-defined margins, the next question
should be: how old is the patient?
Age is the most important clinical clue.
Finally other clues need to be considered, such as a
lesion’s localization within the skeleton and within the
bone, any periosteal reaction, cortical destruction,
matrix calcifications, etc.
īŽ In the table on the left the morphology of a bone
lesion is combined with the age of the patient.
īŽ Notice the following:
īŽ Infections, a common tumor mimic, are seen in any
age group.
īŽ Infection may be well-defined or ill-defined osteolytic,
and even sclerotic.
īŽ EG and infections should be mentioned in the
differential diagnosis of almost any bone lesion in
patients Many sclerotic lesions in patients > 20 years
are healed, previously osteolytic lesions which have
ossified, such as: NOF, EG, SBC, ABC and
chondroblastoma.
īŽ Zone of transition
īŽ In order to classify osteolytic lesions as well-
defined or ill-defined, we need to look at the
zone of transition between the lesion and the
adjacent normal bone.
The zone of transition is the most reliable
indicator in determining whether an osteolytic
lesion is benign or malignant (1).
The zone of transition only applies to
osteolytic lesions since sclerotic lesions
usually have a narrow transition zone.
īŽ Small zone of transition
A small zone of transition results in a sharp, well-defined border
and is a sign of slow growth.
A sclerotic border especially indicates poor biological activity.
In patients In patients > 30years, and particularly over 40 years,
despite benign radiographic features, metastasis or
plasmacytoma also have to be considered
On the left three bone lesions with a narrow zone of transition.
Based on the morphology and the age of the patients, these
lesions are benign.
Notice that in all three patients, the growth plates have not yet
closed.
īŽ In patients > 40 years metastases and
multiple myeloma are the most common bone
tumors.
Metastases under the age of 40 are
extremely rare, unless a patient is known to
have a primary malignancy.
Metastases could be included in the
differential diagnosis if a younger patient is
known to have a malignancy, such as
neuroblastoma, rhabdomyosarcoma or
retinoblastoma.
īŽ Wide zone of transition
An ill-defined border with a broad zone of
transition is a sign of aggressive growth (1).
It is a feature of malignant bone tumors.
There are two tumor-like lesions which may
mimic a malignancy and have to be included
in the differential diagnosis.
These are infections and eosinophilic
granuloma.
Both of these entities may have an
aggressive growth pattern.
īŽ Infections and eosinophilic granuloma are
exceptional because they are benign lesions
which may seem malignant due to their
aggressive biologic behavior.
These lesions may have ill-defined margins,
but cortical destruction and an aggressive
type of periosteal reaction may also be seen.
EG almost always occurs in patients
Infections have to be included in the
differential diagnosis of any bone lesion at
any age.
īŽ Age
īŽ Age is the most important clinical clue in
differentiating possible bone tumors.
There are many ways of splitting age groups, as can
be seen in the first table.
Some prefer to divide patients into two age groups:
30 years.
Most primary bone tumors are seen in patients In
patients > 30 years we must always include
metastases and myeloma in the differential
diagnosis.
īŽ Periosteal reaction
īŽ A periosteal reaction is a non-specific reaction and
will occur whenever the periosteum is irritated by a
malignant tumor, benign tumor, infection or trauma.
There are two patterns of periosteal reaction: a
benign and an aggressive type.
The benign type is seen in benign lesions such as
benign tumors and following trauma.
An aggressive type is seen in malignant tumors, but
also in benign lesions with aggressive behavior, such
as infections and eosinophilic granuloma.
īŽ Benign periosteal reaction
Detecting a benign periosteal reaction may
be very helpful, since malignant lesions never
cause a benign periosteal reaction.
A benign type of periosteal reaction is a thick,
wavy and uniform callus formation resulting
from chronic irritation.
In the case of benign, slowly growing lesions,
the periosteum has time to lay down thick
new bone and remodel it into a more normal-
appearing cortex.
īŽ Aggressive periosteal reaction
This type of periostitis is multilayered, lamellated or
demonstrates bone formation perpendicular to the
cortical bone.
It may be spiculated and interrupted - sometimes
there is a Codman's triangle.
A Codman's triangle refers to an elevation of the
periosteum away from the cortex, forming an angle
where the elevated periosteum and bone come
together.
In aggressive periostitis the periosteum does not
have time to consolidate.
īŽ Aggressive periosteal reaction (2)
īŽ left:
Osteosarcoma with interrupted periosteal rection and Codman's
triangle proximally.
There is periosteal bone formation perpendicular to the cortical
bone and extensive bony matrix formation by the tumor itself.
īŽ middle:
Ewing sarcoma with lamellated and focally interrupted periosteal
reaction. (blue arrows)
īŽ right:
Infection with a multilayered periosteal reaction.
Notice that the periostitis is aggressive, but not as aggressive as
in the other two cases.
īŽ Cortical destruction
īŽ Cortical destruction is a common finding, and not very useful in
distinguishing between malignant and benign lesions.
Complete destruction may be seen in high-grade malignant
lesions, but also in locally aggressive benign lesions like EG and
osteomyelitis.
More uniform cortical bone destruction can be found in benign
and low-grade malignant lesions.
Endosteal scalloping of the cortical bone can be seen in benign
lesions like FD and low-grade chondrosarcoma.
īŽ The images on the left show irregular cortical destruction in an
osteosarcoma (left) and cortical destruction with aggressive
periosteal reaction in Ewing's sarcoma.
īŽ Ballooning is a special type of cortical destruction.
In ballooning the destruction of endosteal cortical bone and the
addition of new bone on the outside occur at the same rate,
resulting in expansion.
This 'neocortex' can be smooth and uninterrupted, but may also
be focally interrupted in more aggressive lesions like GCT.
īŽ left: Chondromyxoid fibroma
A benign, well-defined, expansile lesion with regular destruction
of cortical bone and a peripheral layer of new bone.
īŽ right: Giant cell tumor
A locally aggressive lesion with cortical destruction, expansion
and a thin, interrupted peripheral layer of new bone.
Notice the wide zone of transition towards the marrow cavity,
which is a sign of aggressive behavior.
īŽ Cortical destruction (3)
In the group of malignant small round cell tumors
which include Ewing's sarcoma, bone lymphoma and
small cell osteosarcoma, the cortex may appear
almost normal radiographically, while there is
permeative growth throughout the Haversian
channels.
These tumors may be accompanied by a large soft
tissue mass while there is almost no visible bone
destruction.
The image on the left shows an Ewing's sarcoma with
permeative growth through the Haversian channels
accompanied by a large soft tissue mass.
The radiograph does not shown any signs of cortical
destruction.
īŽ Location within the skeleton
The location of a bone lesion within the skeleton can be a clue in the differential diagnosis.
The illustration on the left shows the preferred locations of the most common bone tumors.
In some locations, such as in the humerus or around the knee, almost all bone tumors may be found.
īŽ Top five location of bone tumors in alphabethic order
īŽ Aneurysmal Bone Cyst
tibia, femur, fibula, spine, humerus
īŽ Adamantinoma
tibia shaft, mandible
īŽ Chondroblastoma
femur, humerus, tibia, tarsal bone (calc), patella
īŽ Chondromyxoid fibroma
tibia, femur, tarsal bone, phalanx foot, fibula
īŽ Chondrosarcoma
femur, rib, iliac bone, humerus, tibia
īŽ Chordoma
sacrococcygeal, spheno-occipital, cervical, lumbar, thoracic
īŽ Eosinophilic Granuloma
femur, skull, iliac bone, rib, vertebra
īŽ Enchondroma
phalanges of hands and feet, femur, humerus, metacarpals, rib
īŽ Ewing's sarcoma
femur, iliac bone, fibula, rib, tibia
īŽ Fibrous dysplasia
femur, tibia, rib, skull, humerus
īŽ Giant Cell Tumor
femur, tibia, fibula, humerus, distal radius
īŽ Hemangioma
spine, ribs, craniofacial bones, femur, tibia
īŽ Lymphoma
femur, tibia, humerus, iliac bone, vertebra
īŽ Metastases
vertebrae, ribs, pelvis, femur, humerus
īŽ Non Ossifying Fibroma
tibia, femur, fibula, humerus
īŽ Osteoid osteoma
femur, tibia, spine, tarsal bone, phalanx
īŽ Osteoblastoma
spine, tarsal bone (calc), femur, tibia, humerus
īŽ Osteochondroma
femur, humerus, tibia, fibula, pelvis
īŽ Osteomyelitis
femur, tibia, humerus, fibula, radius
īŽ Osteosarcoma
femur, tibia, humerus, fibula, iliac bone
īŽ Solitary Bone Cyst
proximal humerus, proximal femur, calcaneal bone, iliac bone
īŽ Location: epiphysis - metaphysis - diaphysis
īŽ Epiphysis
Only a few lesions are located in the epiphysis, so this could be an important finding.
In young patients it is likely to be either a chondroblastoma or an infection.
In patients over 20, a giant cell tumor has to be included in the differential diagnosis.
In older patients a geode, i.e. degenerative subchondral bone cyst must be added to the
differential diagnosis.
Look carefully for any signs of arthrosis.
īŽ Metaphysis
NOF, SBC, CMF, Osteosarcoma, Chondrosarcoma, Enchondroma and infections.
īŽ Diaphysis
Ewing's sarcoma, SBC, ABC, Enchondroma, Fibrous dysplasia and Osteoblastoma.
īŽ
Differentiating between a diaphyseal and a metaphyseal location is not always possible.
Many lesions can be located in both or move from the metaphysis to the diaphysis during
growth.
Large lesions tend to expand into both areas.
īŽ Location: centric - eccentric - juxtacortical
īŽ Centric in long bone
SBC, eosinophilic granuloma, fibrous dysplasia, ABC and enchondroma are lesions that are
located centrally within long bones.
īŽ Eccentric in long bone
Osteosarcoma, NOF, chondroblastoma, chondromyxoid fibroma, GCT and osteoblastoma
are located eccentrically in long bones.
īŽ Cortical
Osteoid osteoma is located within the cortex and needs to be differentiated from
osteomyelitis.
īŽ Juxtacortical
Osteochondroma. The cortex must extend into the stalk of the lesion.
Parosteal osteosarcoma arises from the periosteum.
īŽ
īŽ SBC: central diaphyseal
īŽ NOF: eccentric metaphyseal
īŽ SBC: central diaphyseal
īŽ Osteoid osteoma: cortical
īŽ Degenerative subchondral cyst: epiphyseal
īŽ ABC: centric diaphyseal
īŽ Matrix
īŽ Calcifications or mineralization within a bone lesion may be an
important clue in the differential diagnosis.
There are two kinds of mineralization: a chondroid matrix in
cartilaginous tumors like enchondromas and chondrosarcomsa
and an osteoid matrix in osseus tumors like osteoid osteomas
and osteosarcomas.
Chondroid matrix
Calcifications in chondroid tumors have many descriptions:
rings-and-arcs, popcorn, focal stippled or flocculent.
īŽ left: Enchondroma, the most commonly encountered lesion of
the phalanges.
īŽ middle: middle: Peripheral chondrosarcoma, arising from an
osteochondroma (exostosis).
īŽ right: Chondrosarcoma of the rib.
īŽ Osteoid matrix
Mineralization in osteoid tumors can be described as
a trabecular ossification pattern in benign bone-
forming lesions and as a cloud-like or ill-defined
amorphous pattern in osteosarcomas.
Sclerosis can also be reactive, e.g. in Ewing’s
sarcoma or lymphoma.
īŽ left
Cloud-like bone formation in osteosarcoma.
Notice the aggressive, interrupted periosteal reaction
(arrows).
īŽ right
Trabecular ossification pattern in osteoid osteoma.
Notice osteolytic nidus (arrow).
īŽ Polyostotic or multiple lesions
īŽ Most bone tumors are solitary lesions.
If there are multiple or polyostotic lesions, the differential diagnosis
must be adjusted.
īŽ Polyostotic lesions
NOF, fibrous dysplasia, multifocal osteomyelitis, enchondromas,
osteochondoma, leukemia and metastatic Ewing' s sarcoma.
Multiple enchondromas are seen in Morbus Ollier.
Multiple enchondromas and hemangiomas are seen in Maffucci's
syndrome.
īŽ Polyostotic lesions > 30 years
Common: Metastases, multiple myeloma, multiple enchondromas.
Less common: Fibrous dysplasia, Brown tumors of
hyperparathyroidism, bone infarcts.
īŽ Mnemonic for multiple oseolytic lesions: FEEMHI:
Fibrous dysplasia, enchondromas, EG, Mets and myeloma,
Hyperparathyroidism, Infection.
īŽ Hemangioma.
īŽ Metastasis.
īŽ Multiple myeloma.
īŽ Plasmocytoma: vertebra plana.
This 'Mini Brain' appearance of
plasmacytoma in the spine is sufficiently
pathognomonic to obviate biopsy (9).
Pattern
matrix
Periosteal reaction
Location
18 years old male
īŽ Conventional ~75%
īŽ 10-25yr; smaller peak @ 60s
īŽ metaphysis; distal Femur, prox. Tibia
īŽ osteoid matrix/sclerotic appearance,
aggressive periosteal reaction, soft tissue
mass, NOT usually expansile.
īŽ Most common mets to lung
Osteosarcoma
īŽ Almost considered a
subtype
īŽ occurs later in life than
conventional
īŽ similar appearance
īŽ histologically, more
differentiated
Osteosarcoma of the jaw
26 years old female
īŽ Peaks later ~2nd decade
īŽ metaphysis; posterior distal Femur
īŽ lobulated ossified mass, thickened cortex,
lucent line persists b/t cortex/mass
īŽ Ddx: osteochondroma, myositis ossificans,
periosteal osteosarcoma
īŽ best prognosis of osteosarcomas
Parosteal osteosarcoma (4%)
6 years old girl
īŽ Same age, location as conventional
īŽ usually osteolytic, or expansile; can be purely
lytic
īŽ +/- periosteal reaction or soft tissue involved
īŽ Ddx: Ewing’s, fibrosarcoma/MFH, Giant cell
tumor, and aneurysmal bone cyst
Telangiectatic (3%)
11 years old female
īŽ 10-20 years old
īŽ diaphysis anterior femur; tibia
īŽ spiculated calcified mass, less dense and
solid than parosteal, cortex always involved,
periosteal reaction may be present
īŽ Usually NO medullary destruction
īŽ NO lucent line as in parosteal
Periosteal (1%)
27 years old female
īŽ Same age, location as conventional
īŽ medullary/central location, large sclerotic
lesion +/- lytic components
īŽ Usually NO periosteal reaction or soft tissue
mass
īŽ Ddx: fibrous dysplasia, Giant cell, Ewing’s,
osteomyelitis, lymphoma, MFH
Low grade intraosseous (1%)
īŽ Most cases in
childhood, high grade
īŽ lesions are bilateral and
symmetrical
īŽ appearance similar to
conventional
īŽ metastasis to lungs
early
Multicentric
īŽ Similar to periosteal
type, except high grade
īŽ surface lesion with
destruction and usually
a soft tissue mass
īŽ worse prognosis of
surface types
High Grade surface
īŽ Similar to Ewing’s
histologically except
with osteoid
īŽ permeative bone
destruction, periostitis,
soft tissue mass
īŽ femur, humerus
īŽ worse prognosis than
conventional
Small cell
66 years old woman
īŽ Most commonly occurs in Paget’s,this
represents ~ 3% of osteosarcomas
īŽ <1% Paget’s have malignant degeneration
īŽ other associations include post irradiated
bone
īŽ osteosarcoma in fibrous dysplasia and bone
infarct is extremely rare
Osteosarcoma in abnormal bone
45 years old male
īŽ Peak 30-60 years old, median @45
īŽ metaphysis; femur, humerus,75% near trunk
īŽ scattered chondroid matrix either dense or
light, expansile, can be well defined
depending on grade
īŽ difficult to differentiate from enchondroma
īŽ metastasis is late, may invade veins
Chondrosarcoma
35 years old female
īŽ Average age lower than conventional
īŽ more frequent in multiple exostoses or
enchondromas than single
īŽ Maffucci syndrome, Ollier disease, hereditary
multiple exostosis syndrome
īŽ check for bone destruction, change in
calcification pattern, lesion enlarging
Peripheral chondrosarcoma
32 years old female
īŽ Resembles chondroblastoma
īŽ peak age is higher, 3rd decade
īŽ epiphysis, proximal femur 50%
īŽ lytic, sclerosis at periphery, expansile, 25%
have calcification
īŽ Ddx: chondroblastoma, giant cell, mets,
aneurysmal bone cyst, plasmacytoma
Clear cell
īŽ Varied appearance;
lytic, irregular margins
bone sclerosis
īŽ femur (15%), ribs,
vertebrae
īŽ arise from bone or soft
tissue
īŽ aggressive, poorer
prognosis
Mesenchymal
īŽ Similar to periosteal
osteosarcoma, some
believe it’s the same
lesion, histo diagnosis
īŽ usually presents as soft
tissue mass
īŽ surface lesion with
spotty calcification
Periosteal chondrosarcoma
68 years old female
īŽ Most common primary bone tumor
īŽ peak age 50-80, 98% > 40
īŽ vertebrae 50%, ribs, skull, pelvis
īŽ lytic, punched out lesions, endosteal
scalloping
īŽ difficult differentiate comp. fractures
īŽ death due to pneumonia, renal failure
Multiple myeloma
12 years old female
īŽ Range 5-30, peak age 15
īŽ diaphysis; femur, tibia, pelvis
īŽ permeative, lytic lesion; can be very subtle,
laminated periosteal reaction, NO matrix only
reactive changes
īŽ small cell tumor similar to PNET,lymphoma
īŽ highly aggressive, metastasize to lung, bone
Ewing’s sarcoma
45 years male
īŽ Range 20-60; peak age 41
īŽ meta/diaphysis; femur, tibia
īŽ expansile, lytic or permeative; the later
confers worse prognosis
īŽ usually NO reactive bone formation or
periosteal reaction
īŽ lesions differentiated histologically
Fibrosarcoma / MFH
37 years old female
īŽ 20-40 years old, usually after physis closure
īŽ metaphysis around knee, distal radius
īŽ radiolucent, expansile, eccentric location
īŽ radiographic features do not predict
behavior, up to 20% malignant
īŽ Ddx: chondroblastoma, aneurysmal cyst,
mets, brown tumor, fibrous dysplasia
Giant cell tumor
61 year old female
īŽ Broad age range, peak in 6, 7th decade
īŽ femur (25%) however can be multiple and
involve any bone
īŽ usually permeative pattern,yet can be mixed,
associated soft tissue mass, NO or little
periosteal reaction
īŽ Ddx: mets, Ewing’s, ostoemyelitis
Lymphoma
45 year old female
īŽ Age range 30-70
īŽ sacrococcygeal, spheno-occipital
īŽ derives notochordal remnants or rests
īŽ osteolysis +/- calcification, cortical
destruction, and soft tissue mass
īŽ distant mets rare
Chordoma
33 year old female
īŽ 20-40 years old, usually h/o trauma
īŽ mid diaphysis, tibia > 90%
īŽ eccentric, lytic with sclerotic marigins, may
have satellite lesions
īŽ unclear pathogenesis
īŽ Ddx: other vascular lesions, fibrous dysplasia,
ossifying fibroma
adamantinoma
35 year old female
īŽ Angiosarcoma; all ages, peak 3-4th decade
īŽ meta/diphysis long bones, lower extremity
īŽ appear as multiple, multifocal well defined
osteolytic lesions usually no periosteal
īŽ sometimes associated with chronic
osteomyelitis, osteonecrosis
īŽ local pain and swelling
Hemangioendothelioma
44 year old female
īŽ Hemangiopericytoma
īŽ Liposarcoma, tibia
Other extremely rare tumors

More Related Content

Similar to MSK.ppt

Ankle fractures in children 2021
Ankle fractures in children 2021Ankle fractures in children 2021
Ankle fractures in children 2021GastonDaguerre1
 
Radiology 5th year, 1st lecture (Dr. Salah Mohammad Fatih)
Radiology 5th year, 1st lecture (Dr. Salah Mohammad Fatih)Radiology 5th year, 1st lecture (Dr. Salah Mohammad Fatih)
Radiology 5th year, 1st lecture (Dr. Salah Mohammad Fatih)College of Medicine, Sulaymaniyah
 
Cervical spondylosis
Cervical spondylosisCervical spondylosis
Cervical spondylosisranjan mishra
 
Dr.salah.radiology.radiological approach to bone diseases
Dr.salah.radiology.radiological approach to bone diseasesDr.salah.radiology.radiological approach to bone diseases
Dr.salah.radiology.radiological approach to bone diseasesabas_lb
 
Vgt voorjaar2010 kind_deel2
Vgt voorjaar2010 kind_deel2Vgt voorjaar2010 kind_deel2
Vgt voorjaar2010 kind_deel2vgtrad
 
Galeazzi Fracture
Galeazzi FractureGaleazzi Fracture
Galeazzi Fracturejfreshour
 
Giant cell tumor
Giant cell tumorGiant cell tumor
Giant cell tumorSaurabh Chahar
 
Congenital pseudoarthrosis
Congenital pseudoarthrosisCongenital pseudoarthrosis
Congenital pseudoarthrosisKush Vyas
 
Musculoskeletal imaging rapid review of radiology
Musculoskeletal imaging rapid review of radiologyMusculoskeletal imaging rapid review of radiology
Musculoskeletal imaging rapid review of radiologyDouble M
 
Msk Lecture2 1st Hospital
Msk Lecture2 1st HospitalMsk Lecture2 1st Hospital
Msk Lecture2 1st HospitalSumit Prajapati
 
Presentation1, radiological imaging of anterior knee pain.
Presentation1, radiological imaging of anterior knee pain.Presentation1, radiological imaging of anterior knee pain.
Presentation1, radiological imaging of anterior knee pain.Abdellah Nazeer
 
Cervical Arthritis, Cervical Spondylotic Myelopathy by Pablo Pazmino MD
Cervical Arthritis, Cervical Spondylotic Myelopathy by Pablo Pazmino MDCervical Arthritis, Cervical Spondylotic Myelopathy by Pablo Pazmino MD
Cervical Arthritis, Cervical Spondylotic Myelopathy by Pablo Pazmino MDPablo Pazmino
 
Cervical Arthritis / Cervical Spondylotic Myelopathy / Cervical Stenosis by P...
Cervical Arthritis / Cervical Spondylotic Myelopathy / Cervical Stenosis by P...Cervical Arthritis / Cervical Spondylotic Myelopathy / Cervical Stenosis by P...
Cervical Arthritis / Cervical Spondylotic Myelopathy / Cervical Stenosis by P...Pablo Pazmino
 
Ortho%20downstairs
Ortho%20downstairsOrtho%20downstairs
Ortho%20downstairsjaxemergency
 
Simple and aneurysmal Bone cyst - Definition, Classfication, Investigations, ...
Simple and aneurysmal Bone cyst - Definition, Classfication, Investigations, ...Simple and aneurysmal Bone cyst - Definition, Classfication, Investigations, ...
Simple and aneurysmal Bone cyst - Definition, Classfication, Investigations, ...orthoslides
 
Seropositive arthritis Rheumatoid and others
Seropositive arthritis Rheumatoid and othersSeropositive arthritis Rheumatoid and others
Seropositive arthritis Rheumatoid and othersArif S
 
23205008
2320500823205008
23205008radgirl
 
Mm and pagets
Mm and pagetsMm and pagets
Mm and pagetsBajanagaraju
 

Similar to MSK.ppt (20)

Ankle fractures in children 2021
Ankle fractures in children 2021Ankle fractures in children 2021
Ankle fractures in children 2021
 
Radiology 5th year, 1st lecture (Dr. Salah Mohammad Fatih)
Radiology 5th year, 1st lecture (Dr. Salah Mohammad Fatih)Radiology 5th year, 1st lecture (Dr. Salah Mohammad Fatih)
Radiology 5th year, 1st lecture (Dr. Salah Mohammad Fatih)
 
Cervical spondylosis
Cervical spondylosisCervical spondylosis
Cervical spondylosis
 
Dr.salah.radiology.radiological approach to bone diseases
Dr.salah.radiology.radiological approach to bone diseasesDr.salah.radiology.radiological approach to bone diseases
Dr.salah.radiology.radiological approach to bone diseases
 
Pott Disease
Pott DiseasePott Disease
Pott Disease
 
Vgt voorjaar2010 kind_deel2
Vgt voorjaar2010 kind_deel2Vgt voorjaar2010 kind_deel2
Vgt voorjaar2010 kind_deel2
 
Galeazzi Fracture
Galeazzi FractureGaleazzi Fracture
Galeazzi Fracture
 
Giant cell tumor
Giant cell tumorGiant cell tumor
Giant cell tumor
 
Congenital pseudoarthrosis
Congenital pseudoarthrosisCongenital pseudoarthrosis
Congenital pseudoarthrosis
 
Musculoskeletal imaging rapid review of radiology
Musculoskeletal imaging rapid review of radiologyMusculoskeletal imaging rapid review of radiology
Musculoskeletal imaging rapid review of radiology
 
Msk Lecture2 1st Hospital
Msk Lecture2 1st HospitalMsk Lecture2 1st Hospital
Msk Lecture2 1st Hospital
 
Unusual_Osteoblastoma_of_the_First_Metatarsal_Bone..pdf
Unusual_Osteoblastoma_of_the_First_Metatarsal_Bone..pdfUnusual_Osteoblastoma_of_the_First_Metatarsal_Bone..pdf
Unusual_Osteoblastoma_of_the_First_Metatarsal_Bone..pdf
 
Presentation1, radiological imaging of anterior knee pain.
Presentation1, radiological imaging of anterior knee pain.Presentation1, radiological imaging of anterior knee pain.
Presentation1, radiological imaging of anterior knee pain.
 
Cervical Arthritis, Cervical Spondylotic Myelopathy by Pablo Pazmino MD
Cervical Arthritis, Cervical Spondylotic Myelopathy by Pablo Pazmino MDCervical Arthritis, Cervical Spondylotic Myelopathy by Pablo Pazmino MD
Cervical Arthritis, Cervical Spondylotic Myelopathy by Pablo Pazmino MD
 
Cervical Arthritis / Cervical Spondylotic Myelopathy / Cervical Stenosis by P...
Cervical Arthritis / Cervical Spondylotic Myelopathy / Cervical Stenosis by P...Cervical Arthritis / Cervical Spondylotic Myelopathy / Cervical Stenosis by P...
Cervical Arthritis / Cervical Spondylotic Myelopathy / Cervical Stenosis by P...
 
Ortho%20downstairs
Ortho%20downstairsOrtho%20downstairs
Ortho%20downstairs
 
Simple and aneurysmal Bone cyst - Definition, Classfication, Investigations, ...
Simple and aneurysmal Bone cyst - Definition, Classfication, Investigations, ...Simple and aneurysmal Bone cyst - Definition, Classfication, Investigations, ...
Simple and aneurysmal Bone cyst - Definition, Classfication, Investigations, ...
 
Seropositive arthritis Rheumatoid and others
Seropositive arthritis Rheumatoid and othersSeropositive arthritis Rheumatoid and others
Seropositive arthritis Rheumatoid and others
 
23205008
2320500823205008
23205008
 
Mm and pagets
Mm and pagetsMm and pagets
Mm and pagets
 

Recently uploaded

APM Welcome, APM North West Network Conference, Synergies Across Sectors
APM Welcome, APM North West Network Conference, Synergies Across SectorsAPM Welcome, APM North West Network Conference, Synergies Across Sectors
APM Welcome, APM North West Network Conference, Synergies Across SectorsAssociation for Project Management
 
Arihant handbook biology for class 11 .pdf
Arihant handbook biology for class 11 .pdfArihant handbook biology for class 11 .pdf
Arihant handbook biology for class 11 .pdfchloefrazer622
 
The basics of sentences session 2pptx copy.pptx
The basics of sentences session 2pptx copy.pptxThe basics of sentences session 2pptx copy.pptx
The basics of sentences session 2pptx copy.pptxheathfieldcps1
 
Presiding Officer Training module 2024 lok sabha elections
Presiding Officer Training module 2024 lok sabha electionsPresiding Officer Training module 2024 lok sabha elections
Presiding Officer Training module 2024 lok sabha electionsanshu789521
 
How to Make a Pirate ship Primary Education.pptx
How to Make a Pirate ship Primary Education.pptxHow to Make a Pirate ship Primary Education.pptx
How to Make a Pirate ship Primary Education.pptxmanuelaromero2013
 
CARE OF CHILD IN INCUBATOR..........pptx
CARE OF CHILD IN INCUBATOR..........pptxCARE OF CHILD IN INCUBATOR..........pptx
CARE OF CHILD IN INCUBATOR..........pptxGaneshChakor2
 
SOCIAL AND HISTORICAL CONTEXT - LFTVD.pptx
SOCIAL AND HISTORICAL CONTEXT - LFTVD.pptxSOCIAL AND HISTORICAL CONTEXT - LFTVD.pptx
SOCIAL AND HISTORICAL CONTEXT - LFTVD.pptxiammrhaywood
 
BASLIQ CURRENT LOOKBOOK LOOKBOOK(1) (1).pdf
BASLIQ CURRENT LOOKBOOK  LOOKBOOK(1) (1).pdfBASLIQ CURRENT LOOKBOOK  LOOKBOOK(1) (1).pdf
BASLIQ CURRENT LOOKBOOK LOOKBOOK(1) (1).pdfSoniaTolstoy
 
mini mental status format.docx
mini    mental       status     format.docxmini    mental       status     format.docx
mini mental status format.docxPoojaSen20
 
URLs and Routing in the Odoo 17 Website App
URLs and Routing in the Odoo 17 Website AppURLs and Routing in the Odoo 17 Website App
URLs and Routing in the Odoo 17 Website AppCeline George
 
Concept of Vouching. B.Com(Hons) /B.Compdf
Concept of Vouching. B.Com(Hons) /B.CompdfConcept of Vouching. B.Com(Hons) /B.Compdf
Concept of Vouching. B.Com(Hons) /B.CompdfUmakantAnnand
 
Introduction to ArtificiaI Intelligence in Higher Education
Introduction to ArtificiaI Intelligence in Higher EducationIntroduction to ArtificiaI Intelligence in Higher Education
Introduction to ArtificiaI Intelligence in Higher Educationpboyjonauth
 
Q4-W6-Restating Informational Text Grade 3
Q4-W6-Restating Informational Text Grade 3Q4-W6-Restating Informational Text Grade 3
Q4-W6-Restating Informational Text Grade 3JemimahLaneBuaron
 
Contemporary philippine arts from the regions_PPT_Module_12 [Autosaved] (1).pptx
Contemporary philippine arts from the regions_PPT_Module_12 [Autosaved] (1).pptxContemporary philippine arts from the regions_PPT_Module_12 [Autosaved] (1).pptx
Contemporary philippine arts from the regions_PPT_Module_12 [Autosaved] (1).pptxRoyAbrique
 
Industrial Policy - 1948, 1956, 1973, 1977, 1980, 1991
Industrial Policy - 1948, 1956, 1973, 1977, 1980, 1991Industrial Policy - 1948, 1956, 1973, 1977, 1980, 1991
Industrial Policy - 1948, 1956, 1973, 1977, 1980, 1991RKavithamani
 
Solving Puzzles Benefits Everyone (English).pptx
Solving Puzzles Benefits Everyone (English).pptxSolving Puzzles Benefits Everyone (English).pptx
Solving Puzzles Benefits Everyone (English).pptxOH TEIK BIN
 
Presentation by Andreas Schleicher Tackling the School Absenteeism Crisis 30 ...
Presentation by Andreas Schleicher Tackling the School Absenteeism Crisis 30 ...Presentation by Andreas Schleicher Tackling the School Absenteeism Crisis 30 ...
Presentation by Andreas Schleicher Tackling the School Absenteeism Crisis 30 ...EduSkills OECD
 

Recently uploaded (20)

APM Welcome, APM North West Network Conference, Synergies Across Sectors
APM Welcome, APM North West Network Conference, Synergies Across SectorsAPM Welcome, APM North West Network Conference, Synergies Across Sectors
APM Welcome, APM North West Network Conference, Synergies Across Sectors
 
Arihant handbook biology for class 11 .pdf
Arihant handbook biology for class 11 .pdfArihant handbook biology for class 11 .pdf
Arihant handbook biology for class 11 .pdf
 
The basics of sentences session 2pptx copy.pptx
The basics of sentences session 2pptx copy.pptxThe basics of sentences session 2pptx copy.pptx
The basics of sentences session 2pptx copy.pptx
 
Presiding Officer Training module 2024 lok sabha elections
Presiding Officer Training module 2024 lok sabha electionsPresiding Officer Training module 2024 lok sabha elections
Presiding Officer Training module 2024 lok sabha elections
 
How to Make a Pirate ship Primary Education.pptx
How to Make a Pirate ship Primary Education.pptxHow to Make a Pirate ship Primary Education.pptx
How to Make a Pirate ship Primary Education.pptx
 
CARE OF CHILD IN INCUBATOR..........pptx
CARE OF CHILD IN INCUBATOR..........pptxCARE OF CHILD IN INCUBATOR..........pptx
CARE OF CHILD IN INCUBATOR..........pptx
 
SOCIAL AND HISTORICAL CONTEXT - LFTVD.pptx
SOCIAL AND HISTORICAL CONTEXT - LFTVD.pptxSOCIAL AND HISTORICAL CONTEXT - LFTVD.pptx
SOCIAL AND HISTORICAL CONTEXT - LFTVD.pptx
 
BASLIQ CURRENT LOOKBOOK LOOKBOOK(1) (1).pdf
BASLIQ CURRENT LOOKBOOK  LOOKBOOK(1) (1).pdfBASLIQ CURRENT LOOKBOOK  LOOKBOOK(1) (1).pdf
BASLIQ CURRENT LOOKBOOK LOOKBOOK(1) (1).pdf
 
mini mental status format.docx
mini    mental       status     format.docxmini    mental       status     format.docx
mini mental status format.docx
 
URLs and Routing in the Odoo 17 Website App
URLs and Routing in the Odoo 17 Website AppURLs and Routing in the Odoo 17 Website App
URLs and Routing in the Odoo 17 Website App
 
Concept of Vouching. B.Com(Hons) /B.Compdf
Concept of Vouching. B.Com(Hons) /B.CompdfConcept of Vouching. B.Com(Hons) /B.Compdf
Concept of Vouching. B.Com(Hons) /B.Compdf
 
Introduction to ArtificiaI Intelligence in Higher Education
Introduction to ArtificiaI Intelligence in Higher EducationIntroduction to ArtificiaI Intelligence in Higher Education
Introduction to ArtificiaI Intelligence in Higher Education
 
TataKelola dan KamSiber Kecerdasan Buatan v022.pdf
TataKelola dan KamSiber Kecerdasan Buatan v022.pdfTataKelola dan KamSiber Kecerdasan Buatan v022.pdf
TataKelola dan KamSiber Kecerdasan Buatan v022.pdf
 
Model Call Girl in Tilak Nagar Delhi reach out to us at 🔝9953056974🔝
Model Call Girl in Tilak Nagar Delhi reach out to us at 🔝9953056974🔝Model Call Girl in Tilak Nagar Delhi reach out to us at 🔝9953056974🔝
Model Call Girl in Tilak Nagar Delhi reach out to us at 🔝9953056974🔝
 
Q4-W6-Restating Informational Text Grade 3
Q4-W6-Restating Informational Text Grade 3Q4-W6-Restating Informational Text Grade 3
Q4-W6-Restating Informational Text Grade 3
 
Model Call Girl in Bikash Puri Delhi reach out to us at 🔝9953056974🔝
Model Call Girl in Bikash Puri  Delhi reach out to us at 🔝9953056974🔝Model Call Girl in Bikash Puri  Delhi reach out to us at 🔝9953056974🔝
Model Call Girl in Bikash Puri Delhi reach out to us at 🔝9953056974🔝
 
Contemporary philippine arts from the regions_PPT_Module_12 [Autosaved] (1).pptx
Contemporary philippine arts from the regions_PPT_Module_12 [Autosaved] (1).pptxContemporary philippine arts from the regions_PPT_Module_12 [Autosaved] (1).pptx
Contemporary philippine arts from the regions_PPT_Module_12 [Autosaved] (1).pptx
 
Industrial Policy - 1948, 1956, 1973, 1977, 1980, 1991
Industrial Policy - 1948, 1956, 1973, 1977, 1980, 1991Industrial Policy - 1948, 1956, 1973, 1977, 1980, 1991
Industrial Policy - 1948, 1956, 1973, 1977, 1980, 1991
 
Solving Puzzles Benefits Everyone (English).pptx
Solving Puzzles Benefits Everyone (English).pptxSolving Puzzles Benefits Everyone (English).pptx
Solving Puzzles Benefits Everyone (English).pptx
 
Presentation by Andreas Schleicher Tackling the School Absenteeism Crisis 30 ...
Presentation by Andreas Schleicher Tackling the School Absenteeism Crisis 30 ...Presentation by Andreas Schleicher Tackling the School Absenteeism Crisis 30 ...
Presentation by Andreas Schleicher Tackling the School Absenteeism Crisis 30 ...
 

MSK.ppt

  • 1.
  • 2.
  • 3. ACL Tear & Associations īŽ 38 yo M, hurt skiing Initial findings? Justification for further imaging?
  • 4.
  • 5. Additional Imaging Findings on T1 image? Findings? What additional abnormalities are suspected, if any? īŽ T1W1 FSE Fat sat (farther lateral)
  • 6.
  • 7. Findings so far? Any additional concern (s)? Representative Images in the Notch Any other injury you should specifically look for?
  • 8. Additional Findings (separate patient) Summery of all findings & Dx for this case?
  • 9. ACL Tear & Associations
  • 10. īŽ Initial (film) images most importantly showed an avulsion fracture from the lateral portion of the proximal tibia. Marked bone edema at this site was confirmed on the MR images. īŽ This is Segond Fracture, and represents an avulsion of the lateral capsular ligament. It occurs posterior to Gerdy’s tubercle. īŽ It is important to diagnose this fracture because it is associated with an ACL tear in 75-100%, and a meniscal tear in 67%!
  • 11.
  • 12. īŽ Additional MR images demonstrate “Kissing Contusions” in the lateral portions of the distal femur and proximal tibia. Seen in > 50% of ACL tears (specific but not highly sensitive). īŽ Images through intercondylar notch show a large amount of fluid/edema with non-visualization of the ACL: c/w tear. īŽ Other indirect signs (not shown) include anterior displacement of the tibia (anterior drawer sign) and PCL bowing. īŽ The ACL is the most commonly injured of the major ligaments in the knee.
  • 14. īŽ Final images for this case (from a different patient) showed a tear of the medial collateral ligament & the medial meniscus. īŽ In conjunction with an ACL tear, this represents the “Terrible Triad of O’donoghue” īŽ Classically seen as “clipping injury” from football (valgus/abduction type injury)
  • 15. Maisonneuve Fracture īŽ 35 yo Female, s/p fall; ankle pain
  • 16. Findings, Dx? Any Further Imaging Needed/ Recommended?
  • 18. Final Dx? Mechanism of Injury? īŽ Maisonneuve Fracture īŽ 1st set of images showed ankle fractures involving the medial malleolus and posterior tibia. However, the widening between the distal tibia and fibula should also be noted (suggests tear of the interosseus membrane). īŽ Images of the proximal tibia confirm a spiral fracture of the proximal 3rd of the fibula (interosseus membrane is always torn to this point) īŽ Mechanism: Abduction and External rotation (forces talus laterally against the fibula); distal tibiofibular syndesmosis always ruptured īŽ Associated with: rupture of the deltoid ligament, one candidate initially shown only widening of ankle joint space.
  • 19. Sarcoid of Bone īŽ 63 year old Female, Hand pain Findings?
  • 20.
  • 21. Same patient - Foot Findings, DDx?
  • 22. Same patient after 2 yrs. Rx Try to give Dx/ DDx before Continuingâ€Ļ
  • 23. Additional Images Found In Patient’s Javcket- “Classic” Summary of Findings and Final Dx?
  • 24. īŽ Sarcoid of Bone īŽ Initial images showed a reticulated, “lacelike” pattern involving the 2nd and 5th digits of the left hand, and the 3rd and 5th digits of the right hand. Foot image showed well-defined cystic/ lacelike lesions in the distal phalanx of the 1st digit. īŽ Mineralization and joint spaces essentially normal; no erosions īŽ Lesions improved after systemic treatment (MTX & Steroids) īŽ CXR & MRI confirmed sarcoid findings in the chest & brain īŽ Can also have neuropathy-like destruction of distal phalanges (not present in this case) īŽ Osseus involvement seen in 6-20% of sarcoid cases.
  • 25. Case 1 īŽ History īŽ The patient fell on an outstretched hand and now complains of pain at the thumb.
  • 26.
  • 27. īŽ Diagnosis īŽ Gamekeeper’s Thumb or Skier’s Thumb. īŽ Findings īŽ Soft tissue swelling is noted about the base of the thumb. There is a small, mildly displaced fracture fragment from īŽ the ulnar aspect of the base of the proximal phalanx of the first digit.
  • 28. īŽ Discussion īŽ Gamekeeper’s or Skier’s Thumb results from disruption of the ulnar collateral ligament (UCL) secondary to a valgus īŽ stress applied at the first metacarpophalangeal joint. Abduction- stress films of the thumb with UCL disruption show a īŽ first metacarpal-phalangeal angle of greater than 30 degrees. Often there is an associated avulsed bone fragment off īŽ the base of the proximal phalanx as in the current case. īŽ A “Stener Lesion” occurs when the torn end of the UCL becomes displaced superficial to the adductor pollicus īŽ aponeurosis. MRI and ultrasound best evaluate UCL entrapment.
  • 29. Case 2 īŽ History īŽ Pain. Evaluate for possible fracture.
  • 30.
  • 31. īŽ Diagnosis īŽ Polyostotic Fibrous Dysplasia. īŽ Findings īŽ Plain film of the pelvis shows a “ Shepherd’s Crook” deformity of the femoral neck. There is a long segment expansile lesion of the proximal femur with endosteal scalloping and īŽ a ground glass matrix. A second geographic lesion with a thin sclerotic margin, a short transition zone, and a ground glass matrix is noted in the left pelvis.
  • 32. Discussion īŽ Fibrous dysplasia is a developmental dysplasia where bone is displaced by fibrous tissue. Irregularly arrayed trabeculae of immature bone formed by a metaplasia of fibrous īŽ tissue may be present. Fibrous dysplasia may involve one bone (monostotic, 80 percent) or multiple bones (polyostotic, 20 percent). The cause of fibrous dysplasia is unknown īŽ and is not hereditary. īŽ Polyostotic fibrous dysplasia has a predilection for involvement of one side of the body seen in 90 percent of cases. Polyostotic fibrous dysplasia may be associated with īŽ McCune – Albright Syndrome and other endocrine anomalies. Sarcomatous transformation is rare. īŽ Radiographic findings include expansile lesions that may exhibit endosteal scalloping. The replacement of bone by fibrous tissue leads to a “Smokey” or “ground glass” īŽ appearance to the lesions. Some lesions may appear “cystic” with a thin sclerotic rim as in the current case. Bones are prone to fracture. Fractures of the femoral neck may īŽ contribute to the classic “ Shepherd’s Crook” deformity.
  • 33. Case 3 īŽ History īŽ 24 year old male athlete with anterior knee pain.
  • 34.
  • 35. Diagnosis īŽ Jumper’s Knee (Infrapatellar Insertional Tendinosis). Findings īŽ Plain Images show thickening and abnormal high T2 signal in the proximal patellar tendon at its origin on the inferior patella
  • 36. īŽ Discussion īŽ Patellar tendinosis (Jumper’s Knee) is most commonly found in athletes primarily engaged in running and jumping activities. Jumper’s Knee is a chronic functional overload īŽ injury with both micro- and partial macro-tearing of the tendon. MRI demonstrates thickening and high T2 signal with the proximal patellar tendon. Surgery is generally reserved īŽ for those patients with inadequate symptom relief from conservative therapy
  • 37. Case 4 īŽ History īŽ Pain at forefoot
  • 38.
  • 39. īŽ Diagnosis īŽ Freiberg’s Infraction. īŽ Findings īŽ There is flattening, fragmentation and sclerosis involving the third metatarsal head. There is cortical thickening of the adjacent meta-diaphyseal region of the metatarsal. The īŽ adjacent proximal phalangeal exhibits irregularity, broadening and sclerosis at its base.
  • 40. īŽ Discussion īŽ Freiberg’s infraction affects women three to four times more commonly than men and is most commonly seen in adolescent girls ages 13 to 18 years. The second metatarsal īŽ head is most commonly involved and experimental studies have indicated that this digit is most vulnerable secondary to its relative length. The third and fourth metatarsals may īŽ be involved, but the first digit is rarely affected. īŽ Pathogenesis remains speculative but likely involves repetitive trauma as the inciting event. Pathologically, osteonecrosis is present. The radiographic findings are virtually īŽ pathognomonic and consistent with osteonecrosis at a characteristic site.
  • 41. A 52- years female presented with backache
  • 42.
  • 43. īŽ The lesion is largely, but not solely, geographic in that it appears more permeative in the more anterior portion of the vertebral body on the lateral film. It does not have a sclerotic margin and there is not matrix calcification. there has been slightly compression of the vertebral body in both the AP and medial lateral dimension. While this widening could be accounted for the slight collapse of the body, another explanation may be reasonable as well.
  • 44.
  • 45. īŽ Film. 3 īŽ They shows abnormal osseous uptake at several sites, including T11, L2, L5, the right acetabular region, right hip and femoral neck, right radius, and patchy abnormal uptake in the skull. This information is immediately useful in eliminating several entities in our differential diagnosis: chordoma, giant-cell tumor, osteoblastoma, and lymphoma. Of the remaining three, the pattern of bones that is involved is most characteristic of Paget’s disease.
  • 46.
  • 47. īŽ Film4. īŽ This film demostrates that the radius is slightly overgrown and bowed. In addition, there is slight cortical thickening. These are subtle findings, but are so characteristic of Paget’s disease.
  • 48. īŽ We see lytic highly destructive lesions involving the right femoral metadiaphyseal region, both iliac wings, and the left humeral epiphysis. All of the lesions are aggressive, showing a permeative pattern and wide zone of transition without margination. Although of plains films alone in this case are not sufficiently specific to yield the histologic diagnosis, you shows now be able to supply a reasonable differential diagnosis. Remember also that we like to use all of the information supplied by the films, as this patient is female.
  • 49. īŽ Although this is a subtle example of early Paget's disease, it should be remembered that in its early phase Paget's disease can appear lytic and moderately aggressive. This case emphasizes three points. First of all, it was important to categorize the spine lesion as aggressive or moderately aggressive rather than malignant, since the latter categorization would have precluded Paget's disease from the differential diagnosis. Secondly, following the algorithmic approach to the workup paid off in this case as it does in most cases. Obtaining the bone scan and with it the information regarding the polyostotic nature of the lesion limited the differential diagnosis. Finally, the bone scan directs your attention to other areas to be imaged with plain film. This will very often lead to confirmation of a diagnosis. In this case, CT or MR of the original lesion would not have added significantly to our diagnostic capabilities, since they would have confirmed the presence of a moderately aggressive lesion. Following the algorithmic approach saved the patient the time, expense, and discomfort of undergoing these other testing procedures, as well as the discomfort and anxiety of a biopsy.
  • 50.
  • 51.
  • 52. Diagnosis īŽ Introduction of Musculoskeletal Tumors.
  • 53. īŽ The radiologic workup of musculoskeletal tumors can be both cost- efficient and extremely helpful to the referring clinician if one proceeds in a thoughtful and logical manner. The algorithm shown on Diagram 1 is one that we have found useful for aggressive osseous lesions. īŽ Initially, a musculoskeletal tumor should be simply imaged with a plain film. It should be remembered that plain films remain the most reliable imaging method for assessment of both biologic activity and probable histologic diagnosis of an osseous lesion. Although soft tissue involvement by an osseous lesion may be incompletely assessed by plain film, the osseous findings are seen with much better resolution on plain radiographs than with either CT or MR. Plain film therefore is used to arrive at a reasonable differential diagnosis or at least to categorize the lesion as to degree of aggressiveness. In order to do this, we have found it useful to include an assessment of 10 determinants in the description of a tumor. If these determinants are accurately described, the correct diagnosis or at least a limited differential diagnosis usually becomes obvious. These determinants are as follows:
  • 54. īŽ 1. Age of the patient. This can be an extremely important determinant in some lesions in which the age range of occurrence may be quite narrow. For example, malignant osseous lesions in patients under one year of age are usually metastatic neuroblastoma. Malignant osseous lesions in the age range of 1 to 30 are usually osteosarcoma or Ewing's sarcoma. Malignant osseous lesions in the 30- to 60-year range most commonly will be either chondrosarcoma, primary lymphoma, or malignant fibrous histiocytoma, while malignant lesions in the age range over 50 most commonly will be due to metastatic disease or multiple myeloma. Several other osseous lesions have fairly limited age ranges as well. These will be discussed with the individual cases later on in the section.
  • 55. īŽ 2. Soft tissue involvement. Cortical breakthrough of a bone lesion to create a soft tissue mass generally suggests an aggressive lesion. Such soft tissue masses will often distort but not obliterate nearby muscle planes. īŽ 3. Pattern of bone destruction. Common terminology includes the terms "geographic" (well-defined or map-like lesion, the least aggressive pattern), "moth-eaten" (holes, with less well-defined margins, appearing more aggressive), and "permeative" (a poorly demarcated pattern which is often very difficult to visualize and represents a highly aggressive lesion). It is not always easy to differentiate between the moth-eaten and permeative patterns. Furthermore, since both represent an aggressive pattern, it is not necessary to differentiate between the two, and the term permeative should serve well for both. īŽ 4. Size of lesion. Generally, a larger lesion (greater than 5 cm) is more likely to be malignant or aggressive, but there are many exceptions to this statement, and other determinants are generally more important than this one.
  • 56. īŽ 5. Location of the lesion. Three different types of locations should be noted: the particular bone that is involved, the location in a transverse axis, and the location in a longitudinal axis of a long bone. Occasionally the particular bone involved may be important to the diagnosis. One such example is the tibia, which in addition to hosting most tumors that one can think of, also is the most common location for three uncommon tumors, adamantinoma, ossifying fibroma, and chondromyxoid fibroma. Other categorizations of particular bone involvement might be useful such as axial versus appendicular or flat versus tubular bones, with many lesions clearly favoring one over the other. It is worthwhile to categorize a lesion's location in the transverse axis of a tubular bone (central, eccentric, or a cortically-based epicenter). As will be noted in discussions of individual tumors later on, many tumors have very characteristic locations in the transverse axis. Similarly, many tumors have characteristic locations along the long axis of a tubular bone (epiphysis, metaphysis or diaphysis). Therefore, this location should be identified in the description of the osseous lesion as well.
  • 57. īŽ 6. Zone of transition of the lesion from abnormal to normal bone. A wide zone of transition denotes an aggressive lesion, while a narrow zone is a much less aggressive lesion. īŽ 7. Margination of the lesion. A sclerotic margin generally represents a nonaggressive lesion whereas a nonsclerotic margin often represents an aggressive lesion. There are, however, important exceptions to this, including giant-cell tumor and enchondroma. It is generally true that the determinants narrow zone of transition and sclerotic margin occur together in a lesion and suggest that it is nonaggressive. However, these terms are not synonymous. Similarly, the determinants wide zone of transition and nonsclerotic margin usually occur together in a lesion and suggest that it is aggressive. However, occasionally one may see a lesion with a narrow zone of transition but no sclerotic margin. This very unusual combination of determinants is found most commonly in giant-cell tumor and less commonly in plasmacytoma. It is therefore useful to describe these determinants separately.
  • 58. īŽ 8. Presence of visible tumor matrix. The character of any tumor matrix should be described, since it may be tumor specific or may at least allow categorization of a lesion as bone producing versus cartilage producing. In general, aggressive bone-forming tumors produce amorphous osteoid which is often less dense than normal bone. Less aggressive bone- forming tumors produce better organized, denser bone. The matrix of cartilage- producing tumors is usually quite distinctive, appearing stippled and more dense than normal bone.
  • 59. īŽ 9. Host response. An aggressive lesion may not allow a host response, demonstrating cortical destruction and penetration, with or without periosteal reaction. A less aggressive lesion may result in cortical thickening or sclerosis, cortical thinning without reactive bone formation, or cortical expansion. It might be noted that the character of periosteal reaction is not always a reliable sign in determining the aggressiveness of the lesion. However, generally thin linear periosteal reaction is seen in less aggressive lesions while sunburst periosteal reaction is seen in the more aggressive categories.
  • 60. īŽ 10. Polyostotic versus Monostotic. This is the last determinant and might be the most important, since polyostotic lesions automatically restrict the number of disease processes that might be considered. For example, nonaggressive polyostotic lesions should be confined to fibrous dysplasia, Paget's disease, histiocytosis, multiple exostosis, and multiple enchondromatosis. Aggressive polyostotic lesions would be confined to osseous metastases, multiple myeloma, primary bone tumor with osseous metastases, an aggressive phase of Paget's disease, multifocal osteomyelitis, aggressive histiocytosis, and multifocal vascular bone tumors.
  • 61. īŽ The description of these 10 determinants should yield the diagnosis or at least a short differential diagnosis. The individual musculoskeletal tumors often have very characteristic features among these determinants. After several introductory cases, these sets of determinants will be discussed with each individual entity. If one cannot give a diagnosis, it is important to conclude with the observation of whether the lesion is aggressive or nonaggressive rather than malignant or benign. The reasoning here is that some malignant lesions may appear nonaggressive and several benign lesions often appear highly aggressive (especially osteomyelitis and histiocytosis). If one uses the term benign or malignant in one's description, consideration of such lesions which may appear aggressive but act benign often will be precluded.
  • 62. īŽ If the exact diagnosis is not reached after examination of the plain film, one might attempt to place the lesion in one of the following five categories: īŽ 1. An asymptomatic, benign leave-me-alone lesion, which requires no further imaging or attention. An example might be a fibrous cortical defect or classic nonossifying fibroma. īŽ 2. An asymptomatic, almost certainly benign lesion. Such a lesion could be safely followed without further workup and an example might be a large nonossifying fibroma. īŽ 3. A benign symptomatic lesion with a highly probable diagnosis with the region of involvement well seen. Examples of this might be a giant- cell tumor or chondroblastoma. In many cases, one can proceed to definitive treatment without further imaging of these lesions.
  • 63. īŽ 4. A lesion of uncertain diagnosis and a mixture of aggressive and less aggressive features such that benign or malignant status cannot be confidently assessed. Although a good radiologist can attempt to keep the number of lesions assigned to this category small, some lesions truly belong to this category (such as a low-grade intermedullary chondrosarcoma or an aggressive giant-cell tumor) and must be worked up carefully as if they truly belong to the most aggressive category. īŽ 5. An obviously malignant lesion which requires further workup, perhaps for diagnosis but certainly for staging.
  • 64. īŽ The algorithm on Diagram .1 and .1a is for aggressive osseous lesions (category 4 or 5 above). Reasoning for the order of examinations is developed over the next few cases, but note that CT or MR imaging should generally not be performed until late in the workup. īŽ The algorithm for soft tissue musculoskeletal tumors is different (Diagram .2). Radionuclide scans are not useful. Furthermore, MR does not reliably differentiate malignant from benign tumors. Therefore, biopsy might reasonably be performed prior to chest CT to determine whether there is even any need for a metastatic workup.
  • 65. A year old female who appears cachectic and ill. Look at all the images. This is a case of polyostotic aggressive lesions in a child. We see lytic highly destructive lesions involving the right femoral metadiaphyseal Region, both iliac wings, and the left humeral epiphysis. All the lesions are aggressive, showing a permeative pattern and wide zone of transition without margination. Although the plain films alone in this case are not sufficiently specific to yield the histologic diagnosis, you should now be able to supply a reasonable differential diagnosis. remember also that we like to use all of the information supplied by the films, including that on the Name plate. Although it is not included on this film, I will tell you that the patient’s first name is Ebony and she is from New York City. Does this information help you to narrow down the differential diagnosis?
  • 66.
  • 67.
  • 68. īŽ Plain films are the only radiographic workup required in this patient prior to biopsy. CT or MR would not add enough additional information to make their costs worthwhile. Aggressive polyostotic lesions in a 5-year-old most typically represent Ewing's sarcoma with osseous metastases or else metastatic neuroblastoma. However, one must also remember that young patients occasionally do develop multifocal osteomyelitis which can appear quite aggressive; the patient's clinical appearance is that of a systemic illness. In addition, another polyostotic lesion in young patients is eosinophilic granuloma (EG) or the other diseases in the spectrum of histiocytosis, such as Hand- Christian-Schuller disease. EG can occasionally appear this aggressive and definitely deserves to be considered in this case. īŽ There are two unusual features in this case which make it all the more interesting. First of all, the lesion in the humerus is centered in the epiphysis. Epiphyseal lesions are extremely rare and most commonly represent chondroblastomas or osteomyelitis when seen in the skeletally immature patient. This obviously is not a chondroblastoma because of its degree of aggressiveness. One must make the decision to regard the location of this particular lesion as a red herring and not allow this site of lesion to alter the rest of the impression of an aggressive polyostotic lesion.
  • 69. īŽ The second item of interest is the patient's name. The name Ebony gives us the impression that this patient is black. This information helps one narrow the diagnosis down. First of all, it strengthens the possible diagnosis of multifocal osteomyelitis since if this patient has sickle cell anemia, osteomyelitis is a common complication. However, blood tests showed that the patient is not a sickler. The fact that the patient is black also serves to virtually eliminate the possibility that this is a Ewing's sarcoma, since the latter disease is nearly nonexistent in the black population. Therefore, the most likely diagnosis in this case is metastatic neuroblastoma. Appropriate testing was performed and the patient did not have a neuroblastoma. At biopsy, a round-cell lesion was found. It turned out with further testing to be an unusual lymphoma. īŽ Narrowing the case down to this particular diagnosis is probably not possible in this case, but the discussion surrounding it is what makes it interesting. Furthermore, it serves as a reminder that an aggressive lesion in this age group should have all of the round-cell lesions included in the differential diagnosis: metastatic neuroblastoma, Ewing's sarcoma (or other lymphoma), osteomyelitis, and eosinophilic granuloma. This wide spectrum of diseases including both benign and malignant lesions is an important differential diagnosis to remember.
  • 70. īŽ Look first only at film. 1. the radiographic findings are subtle in this case. We see a highly permeative lesion involving the epiphyseal, metaphyseal and proximal diaphyseal portion of the left humerus. This lesion has a wide zone of transition and is not marginated. It has elicited aggressive periosteal reaction both in a linear and spiculated pattern. There is a large soft tissue mass. The most important feature of this lesion is the presence of osteoid tumor matrix both within the bone and, in a highly amorphous pattern, within the soft tissue mass (Film. 1a). What is your diagnosis?
  • 71. The only possible diagnosis for this extremely aggressive lesion which produces tumor osteoid is osteosarcoma. How should workup proceed? Surgical staging must be considered next. An important part of the staging process is determining the presence or absence of metastatic disease. Most musculoskeletal sarcomas are disseminated by the vascular system, and therefore metastatic disease is expected to localize in the lungs and, a little less commonly, in other osseous sites. The algorithm therefore calls for a bone scan to be evaluated next for the presence of metastatic disease. Look next at film. 2. Give the diagnosis and your suggestive for the next item in an orderly staging workup.
  • 72.
  • 73.
  • 74.
  • 75. īŽ Abbreviations used: īŽ ABC = Aneurysmal bone cyst īŽ CMF = Chondromyxoid fibroma īŽ EG = Eosinophilic Granuloma īŽ GCT = Giant cell tumour īŽ FD = Fibrous dysplasia īŽ HPT = Hyperparathyroidism with Brown tumor īŽ NOF = Non Ossifying Fibroma īŽ SBC = Simple Bone Cyst
  • 76.
  • 77. īŽ The most important determinators in the analysis of a potential bone tumor are: īŽ The morphology of the bone lesion on a plain radiograph īŽ Well-defined osteolytic īŽ ill-defined osteolytic īŽ Sclerotic īŽ The age of the patient īŽ It is important to realize that the plain radiograph is the most useful examination for differentiating these lesions. CT and MRI are only helpful in selected cases. īŽ In this article there are links to other articles about bone tumors.
  • 78.
  • 79. īŽ Most bone tumors are osteolytic. The most reliable indicator in determining whether these lesions are benign or malignant is the zone of transition between the lesion and the adjacent normal bone (1). Once we have decided whether a bone lesion is sclerotic or osteolytic and whether it has a well- defined or ill-defined margins, the next question should be: how old is the patient? Age is the most important clinical clue. Finally other clues need to be considered, such as a lesion’s localization within the skeleton and within the bone, any periosteal reaction, cortical destruction, matrix calcifications, etc.
  • 80.
  • 81. īŽ In the table on the left the morphology of a bone lesion is combined with the age of the patient. īŽ Notice the following: īŽ Infections, a common tumor mimic, are seen in any age group. īŽ Infection may be well-defined or ill-defined osteolytic, and even sclerotic. īŽ EG and infections should be mentioned in the differential diagnosis of almost any bone lesion in patients Many sclerotic lesions in patients > 20 years are healed, previously osteolytic lesions which have ossified, such as: NOF, EG, SBC, ABC and chondroblastoma.
  • 82. īŽ Zone of transition īŽ In order to classify osteolytic lesions as well- defined or ill-defined, we need to look at the zone of transition between the lesion and the adjacent normal bone. The zone of transition is the most reliable indicator in determining whether an osteolytic lesion is benign or malignant (1). The zone of transition only applies to osteolytic lesions since sclerotic lesions usually have a narrow transition zone.
  • 83.
  • 84. īŽ Small zone of transition A small zone of transition results in a sharp, well-defined border and is a sign of slow growth. A sclerotic border especially indicates poor biological activity. In patients In patients > 30years, and particularly over 40 years, despite benign radiographic features, metastasis or plasmacytoma also have to be considered On the left three bone lesions with a narrow zone of transition. Based on the morphology and the age of the patients, these lesions are benign. Notice that in all three patients, the growth plates have not yet closed.
  • 85. īŽ In patients > 40 years metastases and multiple myeloma are the most common bone tumors. Metastases under the age of 40 are extremely rare, unless a patient is known to have a primary malignancy. Metastases could be included in the differential diagnosis if a younger patient is known to have a malignancy, such as neuroblastoma, rhabdomyosarcoma or retinoblastoma.
  • 86.
  • 87. īŽ Wide zone of transition An ill-defined border with a broad zone of transition is a sign of aggressive growth (1). It is a feature of malignant bone tumors. There are two tumor-like lesions which may mimic a malignancy and have to be included in the differential diagnosis. These are infections and eosinophilic granuloma. Both of these entities may have an aggressive growth pattern.
  • 88. īŽ Infections and eosinophilic granuloma are exceptional because they are benign lesions which may seem malignant due to their aggressive biologic behavior. These lesions may have ill-defined margins, but cortical destruction and an aggressive type of periosteal reaction may also be seen. EG almost always occurs in patients Infections have to be included in the differential diagnosis of any bone lesion at any age.
  • 89.
  • 90. īŽ Age īŽ Age is the most important clinical clue in differentiating possible bone tumors. There are many ways of splitting age groups, as can be seen in the first table. Some prefer to divide patients into two age groups: 30 years. Most primary bone tumors are seen in patients In patients > 30 years we must always include metastases and myeloma in the differential diagnosis.
  • 91.
  • 92. īŽ Periosteal reaction īŽ A periosteal reaction is a non-specific reaction and will occur whenever the periosteum is irritated by a malignant tumor, benign tumor, infection or trauma. There are two patterns of periosteal reaction: a benign and an aggressive type. The benign type is seen in benign lesions such as benign tumors and following trauma. An aggressive type is seen in malignant tumors, but also in benign lesions with aggressive behavior, such as infections and eosinophilic granuloma.
  • 93.
  • 94. īŽ Benign periosteal reaction Detecting a benign periosteal reaction may be very helpful, since malignant lesions never cause a benign periosteal reaction. A benign type of periosteal reaction is a thick, wavy and uniform callus formation resulting from chronic irritation. In the case of benign, slowly growing lesions, the periosteum has time to lay down thick new bone and remodel it into a more normal- appearing cortex.
  • 95. īŽ Aggressive periosteal reaction This type of periostitis is multilayered, lamellated or demonstrates bone formation perpendicular to the cortical bone. It may be spiculated and interrupted - sometimes there is a Codman's triangle. A Codman's triangle refers to an elevation of the periosteum away from the cortex, forming an angle where the elevated periosteum and bone come together. In aggressive periostitis the periosteum does not have time to consolidate.
  • 96.
  • 97. īŽ Aggressive periosteal reaction (2) īŽ left: Osteosarcoma with interrupted periosteal rection and Codman's triangle proximally. There is periosteal bone formation perpendicular to the cortical bone and extensive bony matrix formation by the tumor itself. īŽ middle: Ewing sarcoma with lamellated and focally interrupted periosteal reaction. (blue arrows) īŽ right: Infection with a multilayered periosteal reaction. Notice that the periostitis is aggressive, but not as aggressive as in the other two cases.
  • 98.
  • 99. īŽ Cortical destruction īŽ Cortical destruction is a common finding, and not very useful in distinguishing between malignant and benign lesions. Complete destruction may be seen in high-grade malignant lesions, but also in locally aggressive benign lesions like EG and osteomyelitis. More uniform cortical bone destruction can be found in benign and low-grade malignant lesions. Endosteal scalloping of the cortical bone can be seen in benign lesions like FD and low-grade chondrosarcoma. īŽ The images on the left show irregular cortical destruction in an osteosarcoma (left) and cortical destruction with aggressive periosteal reaction in Ewing's sarcoma.
  • 100.
  • 101. īŽ Ballooning is a special type of cortical destruction. In ballooning the destruction of endosteal cortical bone and the addition of new bone on the outside occur at the same rate, resulting in expansion. This 'neocortex' can be smooth and uninterrupted, but may also be focally interrupted in more aggressive lesions like GCT. īŽ left: Chondromyxoid fibroma A benign, well-defined, expansile lesion with regular destruction of cortical bone and a peripheral layer of new bone. īŽ right: Giant cell tumor A locally aggressive lesion with cortical destruction, expansion and a thin, interrupted peripheral layer of new bone. Notice the wide zone of transition towards the marrow cavity, which is a sign of aggressive behavior.
  • 102.
  • 103. īŽ Cortical destruction (3) In the group of malignant small round cell tumors which include Ewing's sarcoma, bone lymphoma and small cell osteosarcoma, the cortex may appear almost normal radiographically, while there is permeative growth throughout the Haversian channels. These tumors may be accompanied by a large soft tissue mass while there is almost no visible bone destruction. The image on the left shows an Ewing's sarcoma with permeative growth through the Haversian channels accompanied by a large soft tissue mass. The radiograph does not shown any signs of cortical destruction.
  • 104.
  • 105. īŽ Location within the skeleton The location of a bone lesion within the skeleton can be a clue in the differential diagnosis. The illustration on the left shows the preferred locations of the most common bone tumors. In some locations, such as in the humerus or around the knee, almost all bone tumors may be found. īŽ Top five location of bone tumors in alphabethic order īŽ Aneurysmal Bone Cyst tibia, femur, fibula, spine, humerus īŽ Adamantinoma tibia shaft, mandible īŽ Chondroblastoma femur, humerus, tibia, tarsal bone (calc), patella īŽ Chondromyxoid fibroma tibia, femur, tarsal bone, phalanx foot, fibula īŽ Chondrosarcoma femur, rib, iliac bone, humerus, tibia īŽ Chordoma sacrococcygeal, spheno-occipital, cervical, lumbar, thoracic īŽ Eosinophilic Granuloma femur, skull, iliac bone, rib, vertebra īŽ Enchondroma phalanges of hands and feet, femur, humerus, metacarpals, rib īŽ Ewing's sarcoma femur, iliac bone, fibula, rib, tibia īŽ Fibrous dysplasia femur, tibia, rib, skull, humerus īŽ Giant Cell Tumor femur, tibia, fibula, humerus, distal radius īŽ Hemangioma spine, ribs, craniofacial bones, femur, tibia īŽ Lymphoma femur, tibia, humerus, iliac bone, vertebra īŽ Metastases vertebrae, ribs, pelvis, femur, humerus īŽ Non Ossifying Fibroma tibia, femur, fibula, humerus īŽ Osteoid osteoma femur, tibia, spine, tarsal bone, phalanx īŽ Osteoblastoma spine, tarsal bone (calc), femur, tibia, humerus īŽ Osteochondroma femur, humerus, tibia, fibula, pelvis īŽ Osteomyelitis femur, tibia, humerus, fibula, radius īŽ Osteosarcoma femur, tibia, humerus, fibula, iliac bone īŽ Solitary Bone Cyst proximal humerus, proximal femur, calcaneal bone, iliac bone
  • 106.
  • 107. īŽ Location: epiphysis - metaphysis - diaphysis īŽ Epiphysis Only a few lesions are located in the epiphysis, so this could be an important finding. In young patients it is likely to be either a chondroblastoma or an infection. In patients over 20, a giant cell tumor has to be included in the differential diagnosis. In older patients a geode, i.e. degenerative subchondral bone cyst must be added to the differential diagnosis. Look carefully for any signs of arthrosis. īŽ Metaphysis NOF, SBC, CMF, Osteosarcoma, Chondrosarcoma, Enchondroma and infections. īŽ Diaphysis Ewing's sarcoma, SBC, ABC, Enchondroma, Fibrous dysplasia and Osteoblastoma. īŽ Differentiating between a diaphyseal and a metaphyseal location is not always possible. Many lesions can be located in both or move from the metaphysis to the diaphysis during growth. Large lesions tend to expand into both areas.
  • 108.
  • 109. īŽ Location: centric - eccentric - juxtacortical īŽ Centric in long bone SBC, eosinophilic granuloma, fibrous dysplasia, ABC and enchondroma are lesions that are located centrally within long bones. īŽ Eccentric in long bone Osteosarcoma, NOF, chondroblastoma, chondromyxoid fibroma, GCT and osteoblastoma are located eccentrically in long bones. īŽ Cortical Osteoid osteoma is located within the cortex and needs to be differentiated from osteomyelitis. īŽ Juxtacortical Osteochondroma. The cortex must extend into the stalk of the lesion. Parosteal osteosarcoma arises from the periosteum. īŽ īŽ SBC: central diaphyseal īŽ NOF: eccentric metaphyseal īŽ SBC: central diaphyseal īŽ Osteoid osteoma: cortical īŽ Degenerative subchondral cyst: epiphyseal īŽ ABC: centric diaphyseal
  • 110.
  • 111. īŽ Matrix īŽ Calcifications or mineralization within a bone lesion may be an important clue in the differential diagnosis. There are two kinds of mineralization: a chondroid matrix in cartilaginous tumors like enchondromas and chondrosarcomsa and an osteoid matrix in osseus tumors like osteoid osteomas and osteosarcomas. Chondroid matrix Calcifications in chondroid tumors have many descriptions: rings-and-arcs, popcorn, focal stippled or flocculent. īŽ left: Enchondroma, the most commonly encountered lesion of the phalanges. īŽ middle: middle: Peripheral chondrosarcoma, arising from an osteochondroma (exostosis). īŽ right: Chondrosarcoma of the rib.
  • 112.
  • 113. īŽ Osteoid matrix Mineralization in osteoid tumors can be described as a trabecular ossification pattern in benign bone- forming lesions and as a cloud-like or ill-defined amorphous pattern in osteosarcomas. Sclerosis can also be reactive, e.g. in Ewing’s sarcoma or lymphoma. īŽ left Cloud-like bone formation in osteosarcoma. Notice the aggressive, interrupted periosteal reaction (arrows). īŽ right Trabecular ossification pattern in osteoid osteoma. Notice osteolytic nidus (arrow).
  • 114.
  • 115. īŽ Polyostotic or multiple lesions īŽ Most bone tumors are solitary lesions. If there are multiple or polyostotic lesions, the differential diagnosis must be adjusted. īŽ Polyostotic lesions NOF, fibrous dysplasia, multifocal osteomyelitis, enchondromas, osteochondoma, leukemia and metastatic Ewing' s sarcoma. Multiple enchondromas are seen in Morbus Ollier. Multiple enchondromas and hemangiomas are seen in Maffucci's syndrome. īŽ Polyostotic lesions > 30 years Common: Metastases, multiple myeloma, multiple enchondromas. Less common: Fibrous dysplasia, Brown tumors of hyperparathyroidism, bone infarcts. īŽ Mnemonic for multiple oseolytic lesions: FEEMHI: Fibrous dysplasia, enchondromas, EG, Mets and myeloma, Hyperparathyroidism, Infection.
  • 116.
  • 117. īŽ Hemangioma. īŽ Metastasis. īŽ Multiple myeloma. īŽ Plasmocytoma: vertebra plana. This 'Mini Brain' appearance of plasmacytoma in the spine is sufficiently pathognomonic to obviate biopsy (9).
  • 118.
  • 119.
  • 120.
  • 121.
  • 123. matrix
  • 126. 18 years old male
  • 127. īŽ Conventional ~75% īŽ 10-25yr; smaller peak @ 60s īŽ metaphysis; distal Femur, prox. Tibia īŽ osteoid matrix/sclerotic appearance, aggressive periosteal reaction, soft tissue mass, NOT usually expansile. īŽ Most common mets to lung Osteosarcoma
  • 128.
  • 129.
  • 130. īŽ Almost considered a subtype īŽ occurs later in life than conventional īŽ similar appearance īŽ histologically, more differentiated Osteosarcoma of the jaw
  • 131. 26 years old female
  • 132. īŽ Peaks later ~2nd decade īŽ metaphysis; posterior distal Femur īŽ lobulated ossified mass, thickened cortex, lucent line persists b/t cortex/mass īŽ Ddx: osteochondroma, myositis ossificans, periosteal osteosarcoma īŽ best prognosis of osteosarcomas Parosteal osteosarcoma (4%)
  • 133.
  • 134. 6 years old girl
  • 135. īŽ Same age, location as conventional īŽ usually osteolytic, or expansile; can be purely lytic īŽ +/- periosteal reaction or soft tissue involved īŽ Ddx: Ewing’s, fibrosarcoma/MFH, Giant cell tumor, and aneurysmal bone cyst Telangiectatic (3%)
  • 136. 11 years old female
  • 137. īŽ 10-20 years old īŽ diaphysis anterior femur; tibia īŽ spiculated calcified mass, less dense and solid than parosteal, cortex always involved, periosteal reaction may be present īŽ Usually NO medullary destruction īŽ NO lucent line as in parosteal Periosteal (1%)
  • 138. 27 years old female
  • 139. īŽ Same age, location as conventional īŽ medullary/central location, large sclerotic lesion +/- lytic components īŽ Usually NO periosteal reaction or soft tissue mass īŽ Ddx: fibrous dysplasia, Giant cell, Ewing’s, osteomyelitis, lymphoma, MFH Low grade intraosseous (1%)
  • 140. īŽ Most cases in childhood, high grade īŽ lesions are bilateral and symmetrical īŽ appearance similar to conventional īŽ metastasis to lungs early Multicentric
  • 141. īŽ Similar to periosteal type, except high grade īŽ surface lesion with destruction and usually a soft tissue mass īŽ worse prognosis of surface types High Grade surface
  • 142. īŽ Similar to Ewing’s histologically except with osteoid īŽ permeative bone destruction, periostitis, soft tissue mass īŽ femur, humerus īŽ worse prognosis than conventional Small cell
  • 143. 66 years old woman
  • 144. īŽ Most commonly occurs in Paget’s,this represents ~ 3% of osteosarcomas īŽ <1% Paget’s have malignant degeneration īŽ other associations include post irradiated bone īŽ osteosarcoma in fibrous dysplasia and bone infarct is extremely rare Osteosarcoma in abnormal bone
  • 145. 45 years old male
  • 146. īŽ Peak 30-60 years old, median @45 īŽ metaphysis; femur, humerus,75% near trunk īŽ scattered chondroid matrix either dense or light, expansile, can be well defined depending on grade īŽ difficult to differentiate from enchondroma īŽ metastasis is late, may invade veins Chondrosarcoma
  • 147.
  • 148. 35 years old female
  • 149. īŽ Average age lower than conventional īŽ more frequent in multiple exostoses or enchondromas than single īŽ Maffucci syndrome, Ollier disease, hereditary multiple exostosis syndrome īŽ check for bone destruction, change in calcification pattern, lesion enlarging Peripheral chondrosarcoma
  • 150.
  • 151. 32 years old female
  • 152. īŽ Resembles chondroblastoma īŽ peak age is higher, 3rd decade īŽ epiphysis, proximal femur 50% īŽ lytic, sclerosis at periphery, expansile, 25% have calcification īŽ Ddx: chondroblastoma, giant cell, mets, aneurysmal bone cyst, plasmacytoma Clear cell
  • 153. īŽ Varied appearance; lytic, irregular margins bone sclerosis īŽ femur (15%), ribs, vertebrae īŽ arise from bone or soft tissue īŽ aggressive, poorer prognosis Mesenchymal
  • 154. īŽ Similar to periosteal osteosarcoma, some believe it’s the same lesion, histo diagnosis īŽ usually presents as soft tissue mass īŽ surface lesion with spotty calcification Periosteal chondrosarcoma
  • 155. 68 years old female
  • 156. īŽ Most common primary bone tumor īŽ peak age 50-80, 98% > 40 īŽ vertebrae 50%, ribs, skull, pelvis īŽ lytic, punched out lesions, endosteal scalloping īŽ difficult differentiate comp. fractures īŽ death due to pneumonia, renal failure Multiple myeloma
  • 157.
  • 158.
  • 159. 12 years old female
  • 160. īŽ Range 5-30, peak age 15 īŽ diaphysis; femur, tibia, pelvis īŽ permeative, lytic lesion; can be very subtle, laminated periosteal reaction, NO matrix only reactive changes īŽ small cell tumor similar to PNET,lymphoma īŽ highly aggressive, metastasize to lung, bone Ewing’s sarcoma
  • 161.
  • 163. īŽ Range 20-60; peak age 41 īŽ meta/diaphysis; femur, tibia īŽ expansile, lytic or permeative; the later confers worse prognosis īŽ usually NO reactive bone formation or periosteal reaction īŽ lesions differentiated histologically Fibrosarcoma / MFH
  • 164.
  • 165. 37 years old female
  • 166. īŽ 20-40 years old, usually after physis closure īŽ metaphysis around knee, distal radius īŽ radiolucent, expansile, eccentric location īŽ radiographic features do not predict behavior, up to 20% malignant īŽ Ddx: chondroblastoma, aneurysmal cyst, mets, brown tumor, fibrous dysplasia Giant cell tumor
  • 167.
  • 168. 61 year old female
  • 169. īŽ Broad age range, peak in 6, 7th decade īŽ femur (25%) however can be multiple and involve any bone īŽ usually permeative pattern,yet can be mixed, associated soft tissue mass, NO or little periosteal reaction īŽ Ddx: mets, Ewing’s, ostoemyelitis Lymphoma
  • 170.
  • 171. 45 year old female
  • 172. īŽ Age range 30-70 īŽ sacrococcygeal, spheno-occipital īŽ derives notochordal remnants or rests īŽ osteolysis +/- calcification, cortical destruction, and soft tissue mass īŽ distant mets rare Chordoma
  • 173. 33 year old female
  • 174. īŽ 20-40 years old, usually h/o trauma īŽ mid diaphysis, tibia > 90% īŽ eccentric, lytic with sclerotic marigins, may have satellite lesions īŽ unclear pathogenesis īŽ Ddx: other vascular lesions, fibrous dysplasia, ossifying fibroma adamantinoma
  • 175. 35 year old female
  • 176. īŽ Angiosarcoma; all ages, peak 3-4th decade īŽ meta/diphysis long bones, lower extremity īŽ appear as multiple, multifocal well defined osteolytic lesions usually no periosteal īŽ sometimes associated with chronic osteomyelitis, osteonecrosis īŽ local pain and swelling Hemangioendothelioma
  • 177. 44 year old female
  • 178. īŽ Hemangiopericytoma īŽ Liposarcoma, tibia Other extremely rare tumors